Pharma 2

You might also like

Download as docx, pdf, or txt
Download as docx, pdf, or txt
You are on page 1of 93

Which of the following controlled drug schedules contains drugs that have no accepted

medical use in the United States?


schedule I

Phase 1 studies are important to:


- determine a safe dosage range of a new drug in humans

Which of the following is not frequently involved in drug oxidations?


- esterases

When added to an intestinal smooth muscle in a tissue bath, two different drugs both
cause relaxation of the muscle but with different EC50's values. Based on this
information, which of the following statements is true?
b) the two drugs have different potencies in causing relaxation

A
Long or continuous exposure of a receptor to an agent that is an antagonist can:
a) result in a phenomenon called supersensitivity
b) desensitize the receptor
c) produce tachyphylaxis
d) cause down regulation of the receptor
e) b and c are correct

D
The term that describes what a drug does to the body and, particularly its mechanism of action,
is called:
a) efficacy
b) pharmacodynamics
c) pharmacokinetics
d) pharmacogenetics
e) pharmacogenomics

E
Which of the following autonomic receptors results in the activation of adenylate cyclase as the
major component of the signaling transduction process?
a) nicotinic
b) muscarinic
c) α1 adrenoceptor
d) α2 adrenoceptor
e) β1 adrenoceptor

B
Bethanechol is administered subcutaneously (sc) to a patient with postoperative abdominal
distention and gastric atony. The sc route is chosen over the oral route because gastric
retention is complete and there is no passage of gastric contents into the duodenum. Which of
the following effects might be observed after the sc administration of bethanechol:
a) skeletal muscle twitching
b) decrease in heart rate
c) brochodilation of the airway in the lung
d) relaxation of the detrusor muscle in the bladder
e) inhibition of the release of nitric oxide from vascular endothelial cells
B
The most toxic substance known that affects the parasympathetic nervous system is an
exotoxin secreted by the anaerobe Clostridium botulinum. There are several forms of botulinum
toxin, all of which are highly toxic after they are ingested in contaminated food. The toxin
produces respiratory paralysis by:
a) blocking nicotinic acetylcholine receptors
b) blocking the release of acetylcholine from nerve endings
c) blocking peristalsis
d) causing circulatory collapse
e) stimulating the vagal nerve

C
The cardiovascular effects of epinephrine in a person treated with phentolamine will most likely
resemble the response after the administration of:
a) phenylephrine
b) terbutaline
c) isoproterenol
d) norepinephrine
e) dobutamine

A 70-year-old male patient comes into your office complaining of difficulty in urinating. He says
he has a poor urine stream, strains to pass urine, has prolonged micturition and a feeling of
incomplete bladder emptying. You diagnose this patient as having benign prostatic hyperplasia.
Which of the following would be the best choice of treatment.
a) bethanechol
b) phentolamine
c) dopamine
d) tamsulosin
e) phenoxybenzamine

D. Tamsulosin is an alpha-1(alpha) blocker used in the treatment of BPH

During a laboratory demonstration to depict the complexity of neurotransmission in autonomic


ganglia, Professor PharmPhys sets up an anesthetized mammalian preparation in which he is
recording postsynaptic events following the electrical stimulation of preganglionic sympathetic
nerves. This demonstrates a complex action potential consisting of a fast EPSP followed by a
slow IPSP, followed by a slow EPSP and finally by a very late and slow EPSP (see figure
below).

In Professor PharmPhy's demonstration, a drug which would block the fast EPSP would be:
a) atropine
b) trimethaphan
c) prazosin
d) propranolol
e) succinylcholine
B. Trimetaphan is a nicotinic blocker. 
Atropine is a M antagonist, Prazosin is a alpha-1 antagonist, Propanolol is a nonselective beta
blocker, Succinylcholine is a nicotinic Ach receptor binder that causes prolonged depolarization.

Error! Filename not specified.

During a laboratory demonstration to depict the complexity of neurotransmission in autonomic


ganglia, Professor PharmPhys sets up an anesthetized mammalian preparation in which he is
recording postsynaptic events following the electrical stimulation of preganglionic sympathetic
nerves. This demonstrates a complex action potential consisting of a fast EPSP followed by a
slow IPSP, followed by a slow EPSP and finally by a very late and slow EPSP (see figure
below).
In Professor PharmPhys's demonstration the mediator of the IPSP is:
a) neuropeptide Y
b) acetylcholine
c) dopamine
d) serotonin
e) angiotensin
C. IPSP is mediated by an intermediate synapse by a SIF (smal intensely flourescent cell).
Usually a catecholamine, usually dopamine, is released by this cell to activate the IPSP, which
modulates ganglionic transmission (9-28-29).
Error! Filename not specified.
Contraction of uterus is through alpha-1. Contraction of vascular smooth muscle is through
alpha-1. Mydriasis is through alpha-1. Platelet aggregation is through alpha-2.
Which of the actions of epinephrine would be antagonized by propranolol but not by
phentolamine
a) release of renin
b) mydriasis
c) contraction of the uterus
d) aggregation of platelets
e) contraction of vascular smooth muscle
A. Renin release is through beta-1

E. Alpha-1 receptors trigger PLC activation and rises in IP3 and DAG (also, M1,3,5 do the
same). Phentolamine is a nonselective alpha blocker, Carvedilol is a nonselective beta-blocker,
Metoprolol is a somewhat selective beta-1 blocker, and Atropine is a muscarinic blocker.
The above figure depicts a signaling pathway for some type of drugs or hormones. If the drug
activating this receptor was norepinephrine, which of the following would be the most selective
antagonist to block the receptor:

a) phentolamine
b) carvedilol
c) metoprolol
d) atropine
e) prazosin
Error! Filename not specified.

E. Phenylephrine is an alpha-1 agonist that would increase SVR and through baroreflex,
decrease HR and thus CO. 
NE would increase SVR but also HR and CO through Beta-1, E would also increase HR and
CO, but not change as much SVR and BP as NE. Clonidine (alpha-2 agonist) would decrease
pressure and SVR. Isoproterenol (beta agonist) would increase HR and CO, while decreasing
diastolic and MAP by a little.
A new drug was given i.v. to 25 normal subjects in a Phase I clinical trial. The cardiovascular
effects are summarized in the table above. Which of the following drugs does the new
experimental drug most resemble?
(Hint: Human Simulation Demonstration)
a) norepinephrine
b) epinephrine
c) clonidine
d) isoproterenol
e) phenylephrine
Error! Filename not specified.
D. Most insecticides are organophosphate irreversible (suicide) cholinesterase blockers.
A farmer is brought into the Emergency Department after an accident with crop spraying
equipment, during which he was copiously sprayed with insecticide. He is salivating profusely
and has difficulty in breathing; his pulse rate is 40 beats/min. It is likely that the insecticide
bound to which of the following receptors or effectors?
a) anionic site of acetylcholinesterase
b) nicotinic cholinergic receptor
c) muscarinic cholinergic receptor
d) esteratic site of acetylcholinesterase
e) alpha 1 adrenoceptor

C, pupillary dilation, which is controlled by alpha-1 receptors.

Tachycardia would beta agonists, bronchodilation would be beta2, vasodilation would be beta2.
Salivation is stimulated by alpha-1 receptors, but would be profuse only if stimulated by
muscarinic parasympathetic receptors.

A child has swallowed the contents of two bottles of a nasal decongestant whose primary
ingredient is a potent alpha adrenoceptor agonist drug. The signs of alpha activation that may
occur in this patient include:
a) bronchodilation
b) tachycardia
c) pupillary dilation
d) vasodilation
e) profuse salivation

C. Methamphetamine increases the release of NE at the postganglionic terminal.

Dobutamine is a beta-1 agonist, phenylephrine is an alpha-1 agonist, prazosin is a alpha-1


blocker, metoprolol is a somewhat selective beta-1 blocker. All of these drugs operate at
adrenergic receptors, not at postganglionic terminal
If the diagram represents a preganglionic and postganglionic sympathetic neuron, a drug that
could act at site 5 is:
a) dobutamine
b) phenylephrine
c) methamphetamine
d) prazosin
e) metoprolol
Error! Filename not specified.

C. At eccrine sweat gland, there are cholinergic muscarinic receptors, although it is controlled
by sympathetic innervation. Scopolamine, a muscarinic antagonist, would block this
transmission.

Phentolamine is an alpha blocker, propranolol is a beta blocker, tamsulosin is an alpha-1


blocker used in BPH, neostigmine is a reversible acetylcholinesterase inhibitor.
If the effector cell in the diagram is a thermoregulatory sweat gland (eccrine), a drug that could
antagonize the transmitter released from site 5 and act at site 6 could be:
a) phentolamine
b) propranolol
c) scopolamine
d) tamsulosin
e) neostigmine
Error! Filename not specified.
B. The neuromuscular junction receptor is nicotinic. Those that control gastric secretion, salivary
glands, sweat glands, and detrusor muscle of the bladder are all muscarinic.

Physostigmine is an AChEI, while bethanechol is a muscarinic agonist. Physostigmine will thus


have action at any cholinergic receptor, muscarinic or nicotinic. Bethanechol will only have
action at muscarinic
A patient requires mild parasympathomimetic stimulation following surgery. Physostigmine and
bethanechol in small doses have significantly different effects on which of the following:

a) gastric secretion
b) neuromuscular junction (skeletal muscle)
c) salivary glands
d) sweat glands
e) detrusor muscle of the bladder
D. 

Atropine and scopolamine are well absorbed from GI tract, but not so much from eye or intact
skin.

Ipratropium (muscarinic antag) has poor systemic absorption, and is usually inhaled for COPD
and asthma.

Physostigmine readily crosses BBB, while neostigmine/pyridostigmine do not cross very well.

Isoproterenol (beta agonist) which is used for heart block and some asthma, is poorly absorbed
after oral administration.

Dobutamine is administered via IV.


Which of the following pairs of drugs and properties is most correct?
a) atropine: poorly absorbed after oral administration
b) ipratropium: well absorbed after oral administration; long elimination half life
c) neostigmine: readily crosses the blood brain barrier
d) isoproterenol: poorly absorbed after oral administration
e) dobutamine: well absorbed after oral administration

B. Carvedilol is nonselective beta blocker, but it also has alpha-1 adrenoceptor antagonist
activity. This would block the BP rise associated with alpha-1 receptor stimulation by epi.
Propanolol, Pindolol, Timolol are all nonselective beta blockers. Metoprolol is a somewhat
selective blocker for beta-1, but will also have activity against beta-2, and is thus not a good
option.
A new drug was administered to an anesthetized animal with the results shown. A large dose of
epinephrine was administered before and after the new agent for comparison. Which of the
following agents does the new drug most closely resemble?

a) propranolol
b) carvedilol
c) pindolol
d) metoprolol
e) timolol
Error! Filename not specified.

E. ???

Which of the following can be blocked by atropine:

a) decreased blood pressure caused by hexamethonium or trimethaphan


b) increased blood pressure caused by nicotine
c) increased skeletal muscle strength caused by ambenonium or neostigmine
d) tachycardia caused by exercise
e) tachycardia caused by infusion of acetylcholine
C. Postural hypotension, partly because muscarinic (parasympathetic) blockade would not
cause hypotension. Muscarinic blockage would cause mydriasis (dilation), tachycardia, blurred
vision (cycloplegia), and dry mouth. (red as beet, blind as bat, dry as bone, hot as hare, mad as
hatter)

The following systems are under predominant parasympathetic tone:


Myocardium (atrium, SA node), iris, ciliary muscle, GI tract, urinary bladder, salivary glands,
sweat glands.

The following systems are predominantly under symp tone: ventricles, blood vessels, arterioles,
veins.
A 30 year old man has been treated with several autonomic drugs for 4 weeks. He is now
admitted to the Emergency Department showing signs of drug toxicity. Which of the following
signs would distinguish between an overdose of a ganglion blocker and a muscarinic blocker:

a) mydriasis
b) tachycardia
c) postural hypotension
d) blurred vision
e) dry mouth, constipation
D. Isoproterenol is a non-selective beta AGONIST that usually increases HR, contractility, CO,
while also causing vasodilation. Systolic BP usually remains unchanged, but diastolic falls
somewhat.
Which of the following drugs most resembles drug X?
a) acetylcholine
b) atropine
c) epinephrine
d) isoproterenol
e) norepinephrine
Error! Filename not specified.
E. Norepinephrine.
Which of the following most resembles drug Y?
a) acetylcholine
b) atropine
c) epinephrine
d) isoproterenol
e) norepinephrine
Error! Filename not specified.
B. Neonates have low levels of glucuronyl transferase, leading to inability to do phase II
biotransformation of drugs such as antibacterial chloramphenicol. This leads to Gray Baby
syndrome.
A neonate has been diagnosed with bacterial meningitis and is being treated with drug X.
During the first 24 hrs the patient suffers from vomiting, irregular and rapid respirations. During
the second day he presents with flaccidity, an ashen-green color, a decrease in body
temperature and is very ill. You correctly surmise that drug X was not being adequately
metabolized by:
a) acetylation
b) glucuronidation
c) CYP450 hydroxylation
d) methylation
e) sulfate conjugation

E. Codeine is metabolized by CYP2D6


A polymorphism in drug biotransformation includes a defect that interferes with the
biotransformation of codeine to morphine. The defect involves which of the following CYP
enzymes?
a) CYP3A4
b) CYP1A2
c) CYP24
d) CYP5
e) CYP2D6
D. CYP3A and glucuronidation

Which of the following combinations of phase 1 and phase 2 biotransformations account for the
majority of drug biotransformations:

a) CYP2D6 and glucuronidation


b) CYP1A2 and acetylation
c) CYP3A and oxidative deamination
d) CYP3A and glucuronidation
e) CYP2E1 and sulfate conjugation
C. The metabolite is almost always pharmacologically inactive.
Which of the following is most true of synthetic drug biotransformation reactions:
a) the type of reaction is relatively unpredictable
b) the metabolite is almost always less polar
c) the metabolite is almost always pharmacological inactive
d) are all carried out by microsomal enzymes
e) the rate of glucuronide conjugation is not stimulated by prior treatment with drugs
C. Physiological antagonism refers to an interaction which results from one drug antagonizing
the response of another drug following stimulation of a different receptor.

This is a type of nonreceptor antagonism, along with chemical antagonism, in which the
antagonist sequesters agonist and prevents its activity.

There is also receptor antagonism, which may be competitive or noncompetitive.


Physiological antagonism refers to:

a) an interaction which results from one drug antagonizing the effect of another by competing for
the same receptor
b) an interaction which results from a drug binding to a receptor and providing a response
c) an interaction which results from one drug antagonizing the response of another drug
following stimulation of a different receptor
d) an interaction which results when one drug changes the absorption, distribution
biotransformation or excretion of another drug
e) a beneficial interaction when two drugs are simultaneously administered

D. Partial agonist.
Timolol (nonselective beta blocker), Labetalol (nonselective beta blocker), and celiprolol (beta-1
blocker) all have partial agonist activity.

In the absence of other drugs or an endogenous agonist, pindolol produces some increase in
heart rate by activating β-adrenoceptors. However, when the heart rate has been increased by
epinephrine, pindolol causes a dose-dependent reversible decrease in heart rate. From this
information you deduce that pindolol is:
a) a non-competitive antagonist
b) a physiological antagonist
c) a chemical antagonist
d) a partial agonist
e) an inverse agonist

B. Tolterodine is an antimuscarinic used to treat urinary incontinence.


Physostigmine is a AChEI that readily crosses BBB. Clonidine is an alpha-2 agonist. Cocaine
blocks catecholamine reuptake. Prazosin is an alpha-1 blocker used in relief of urinary
symptoms caused by BPH.

Which of the following drugs is correctly matched with a receptor with which it can activate or
bind?
a) physostigmine : nicotinic acetylcholine receptor
b) tolterodine : muscarinic acetylcholine receptor
c) clonidine : α1 adrenoceptor
d) cocaine : monoamine oxidase
e) prazosin : α2 adrenoceptor
E. Pindolol. Beta-blockers and calcium channel blockers act to decrease the heart's workload,
and thus its requirement for oxygen. Only agent that would decrease HR and contractility.

Scopolamine is actually a muscarinic antagonist, and would increase HR.

Phentolamine is a alpha blocker that would produce hypotention, but also reflex tachycardia
and arrhythmias that would aggravate angina.

Isoproterenol, a beta agonist, would increase HR and contractility and oxygen demand, and
thus angina.

Phenoxybenzamine (alpha antagonist) would, like phentolamine produce hypotension with


reflex tachycardia.

A normotensive 65-year-old female began experiencing dyspnea upon excretion associated


with chest pain. The chest discomfort was determined to be angina pectoris. In order to improve
exercise tolerance, you prescribed medication. Consider each drug and rationale and select the
most appropriate.

a) scopolamine: the muscarinic agonistic effects would reduce heart rate and therefore reduce
oxygen demand
b) phentolamine: hypotension associated with phentolamine would reduce myocardial work and
prevent ischemic symptoms
c) isoproterenol: increased β1-receptor stimulation would increase cardiac output and therefore
increase coronary perfusion resulting in decreased angina symptoms
d) phenoxybenzamine: because of its long action, phenoxybenzamine is the preferred treatment
for angina. The decrease in blood pressure from α-receptor blockade is long term and
myocardial work is decreased because of reduced afterload
e) pindolol: reduced effects of adrenergic nerve activity and circulating catecholamines on the
heart result in reduced rate and inotropy. Both factors reduce myocardial oxygen demand and
therefore may increase exercise tolerance and reduce anginal events.

D
Administration of a drug such as phenobarbital, which induces the liver microsomal system may
lead to:
a) an increase in CYP450
b) an increase in glucuronide conjugation
c) an increase in amine oxidase
d) both a and b
e) a, b and c

D. Atropine is well absorbed and crosses the BBB. Physostigmine is one of the few
pharmaceutical AChEIs that cross the BBB. It is the drug of choice for atropine poisoning.

Neostigmine and Edrophonium are AChEIs, but they don't cross the BBB. Tolterodine is a
muscarinic antagonist like atropine. Ipratropium is also an antimuscarinic (but is more poorly
absobred, and is used for COPD and asthma)

During the Iraq crisis several people, believing they had been or were about to be exposed to a
nerve gas, mistakenly injected themselves with syrettes containing a total of 10 mg of atropine.
In the absence of a cholinergic agonist, 10 mg is a toxic dose for adults. The most appropriate
drug for treating atropine toxicity in these people is:
a) neostigmine
b) edrophonium
c) tolterodine
d) physostigmine
e) ipratropium

D
The use of the word "cholinergic" to describe the action of agonists at certain sites is intended to
make it clear that these agonists all:
a) work on nicotinic acetylcholine receptors
b) involve the second messenger cyclicAMP in the production of their effects
c) prevent the action of acetylcholine at the effectors of the parasympathetic nervous system
d) have actions like acetylcholine at those sites
e) are esters, as is acetylcholine

D. Prazosin is an alpha-1 antagonist. Epi's pressor effectors mostly come from stimulation of
alpha-1's in vasculature that lead to contraction of smooth muscle and thus increase in vascular
resistance.

Dopamine selectively dilates arterioles in the renal vascular bed.


Epinephrine:

a) selectively dilates arterioles in the renal vascular bed


b) is a bronchodilator which does not increase blood pressure at any dose
c) stimulates the heart but does not cause peripheral vasodilation at any dose
d) causes a pressor effect which is reversed by prazosin
e) is a vasopressor which does not increase the heart rate in someone pretreated with atropine

A. Metaproterenol is a beta-2 agonist used in the treatment of asthma and COPD.

Dobutamine is a beta-1 agonist, metoprolol is a B1>B2 antagonist, timolol is a nonselective beta


antagonist, prazosin is a alpha-1 blocker.
A patient who is a heavy smoker comes into your office with severe breathing problems. The
patient is suffering from chronic bronchitis and emphysema. A good choice for treating this
patient's symptoms would be:
a) metaproterenol
b) dobutamine
c) metoprolol
d) timolol
e) prazosin
D. COPD, because beta-2 receptors control bronchodilation
A beta adrenoceptor antagonist such as propranolol has been found valuable in treating a wide
variety of indications. You would have the greatest concern in patients suffering from:

a) essential hypertension
b) angina pectoris
c) congestive heart failure
d) chronic obstructive pulmonary disease
e) idiopathic hypertrophic subaortic stenosis
E. Muscarinic agonist used in the treatment of glaucoma.
A patient comes into your office with pain in the eye. You determine that the patient's
intraoccular pressure is 80-90 mmHg (normal is 10 mmHg). You decide the patient is suffering
from acute, narrow angle glaucoma and immediately treat with pilocarpine. You later tell your
assistant that pilocarpine:
a) is inactivated by cholinesterase
b) is a cholinesterase inhibitor
c) blocks muscarinic receptors
d) is a beta adrenoceptor antagonist
e) is a directly acting parasympathomimetic
C. Methamphetamine enhances release of vesicles containting NE, which is pointless if
reserpine (an irreversible VMAT antagonist) depleted those vesicles.

Phenylephrine is a alpha-1 agonist, isoproterenol is a beta antagonist, and epi and NE can act
directly as well
An elderly patient was treated for a long period of time with reserpine to control high blood
pressure. The subsequent administration of which of the following would be expected to
produce the least change in blood pressure:

a) phenylephrine
b) isoproterenol
c) methamphetamine
d) epinephrine
e) norepinephrine
B. Underlying disease may compromises biotransformation and excretion.
Adverse drug reactions are more frequently reported in the elderly because:
a) they always require a lower does than do younger adults
b) underlying disease may compromise biotransformation and excretion
c) they have more office visits
d) most elderly patients are depressed
e) decreased intestinal transient time increases drug absorption
D. alpha1- adrenoceptor antagonist would block this compound, which is most likely an alpha-1
agonist.

Alpha-1 agonist alone would increase SVR, while decreasing HR through baroreflexive means,
and would also produce mydriasis because alpha-1 receptors on the radial muscle produce this
response. Scopolamine, a muscarinic blocker, would dampen the parasympathetic
baroresponse and tone, producing an INCREASE in HR, while also dampening muscarinic
receptors on the iris sphincter, thus producing a more aggravated mydriasis.
After medical school you decide to take a job in the pharmaceutical industry. You are assigned
to the Pharmacology Department and are quite happy because you remember all those
wonderful times in the Principles of Pharmacology class in medical school. Your lab has been
given a new compound for screening. Following IV administration, it produced an increase in
mean arterial blood pressure, a decrease in heart rate, and mydriasis. In the presence of
scopolamine the unknown drug increased both blood pressure and heart rate and produced
even greater mydriasis you believe you have correctly categorized this compound and tell your
assistant that is could be blocked by:
a) muscarinic cholinergic antagonist
b) β1-adrenoceptor antagonist
c) α2-adrenoceptor antagonist
d) α1-adrenoceptor antagonist
e) cholinesterase inhibitor
46. C
47. B
48. A
Dose response curves are depicted in the above Figure for a number of drugs A through E in
relationship to drug X (--- x ---). All drugs act by the same mechanism. Select the letter of the
curve that best fits the description. C is one from far right, A is first curve from left.

46. Equiefficacious but less potent than X


47. Equipotent but more efficacious than X
48. More potent but less efficacious than X
Error! Filename not specified.
49. D, an inhibitor of tyrosine hydroxylase, and thus causes decrease of NE and other
neurotransmitters.
50. E, Edrophonium is a AChEI
51. C (or B??), tubocurarine is a nondepolarizing neuromuscular blocker
52. C, terbutaline is a beta-2 agonist
a) Site A
b) Site B
c) Site C
d) Site D
e) Site E
The above diagram depicts targets for drug action (A→E) located in nerves or innervated
organs. Match the following drugs with their primary site of action. (Letter can be used more
than once)
49. α-methylptyrosine
50. edrophonium
51. tubocurarine
52. terbutaline
Error! Filename not specified.
b) "Lily the Pink"
The Theme Song for the Principles of Pharmacology course is:
a) "Danny Boy"
b) "Lily the Pink"
c) "I Want a New Drug"
d) "Belladonna Blues"
e) "I'll Take You Home Again Kathleen"
E. Vd = dose / Cp(0) = 100 mg / (5mg/L) = 20 L
A new drug was studied in 20 healthy volunteers to determine basic pharmacokinetic
parameters. A dose of 100 mg was administered as an intravenous bolus to each volunteer and
blood samples were analyzed at intervals as shown in the graph below. The plasma
concentrations at each time are shown. The volume of distribution of the new drug is
approximately
a) 0.05 L
b) 0.1 L
c) 5 L
d) 10 L
e) 20 L
Error! Filename not specified.
D. 200 mg.
Vd = dose/Cp(0)
200 liters = dose / (1mg/L)
Dose = 200 mg
A narcotics addict is brought to the emergency room in a deep coma. His friends state that he
took a large dose of morphine 6 hours earlier. An immediate blood analysis shows a morphine
blood level of 0.25 mg/L. Assuming that the pharmacokinetics of morphine in this patient are Vd
= 200 liters and half-life is 3 hours, how much morphine did the patient inject 6 hours earlier?
a) 25 mg
b) 50 mg
c) 100 mg
d) 200 mg
e) 400 mg
B. 1 hour.

Cl = (0.7/(half-life))*Vd
5.6 = (0.7/halflife)*8L
half life = (0.7*8)/5.6 = 1 hour
A normal volunteer will receive a new drug in a phase I clinical trial. The clearance and volume
of distribution of the drug in this subject are 5.6 L/hr and 8 L, respectively. The half-life of the
drug in this subject will be approximately
a) 8 hours
b) 1 hour
c) 40 hours
d) 2 hours
e) 0.02 hours
D. Verapamil is an example of a drug that undergoes substantial first-pass elimination, and the
fact that its clearance = liver blood flow is testament to that. Verapamil, ASA, glyceryl trinitrate,
isosorbide dinitrate, levodopa, lidocaine, metoprolol, morphine, propranolol, salbutamol are
other drugs that undergo substantial first pass metabolism.
Verapamil and phenytoin are both eliminated from the body by metabolism in the liver.
Verapamil has a clearance of 1.5 L/min, approximately equal to liver blood flow, whereas
phenytoin has a clearance of 0.1 L/min. When these compounds are administered under
conditions that increase liver blood flow, which of the following is most likely?
a) the clearance of both verapamil and phenytoin will be increased
b) the clearance of both verapamil and phenytoin will be decreased
c) the clearance of verapamil will be unchanged, whereas the clearance of phenytoin will be
increased
d) the clearance of phenytoin will be unchanged, whereas the clearance of verapamil will be
increased
e) the clearance of both verapamil and phenytoin will be unchanged
E. The time it takes for a drug to reach 97% of Css is equal to 5 half lives of that drug. 94% = 4
half lives, 90% = 3.3 half lives, 80% = 3 half lives, 75% = 2 half lives, 50% = 1 half life.
A nineteen-year-old woman is brought to the hospital with severe asthmatic wheezing. You
decide to use IV theophylline for treatment. The pharmacokinetics of theophylline include the
following average parameters: Vd is 35 L; cl is 48 mL/min; half-life 8 is hours. If an IV infusion of
theophylline is started at a rate of 0.48 mg/min, how long will it take to reach 97% of the final
steady state?
a) approximately 48 minutes
b) approximately 5.8 hours
c) approximately 6 hours
d) approximately 8 hours
e) approximately 40 hours
C
The figure above depicts the drug concentration curves vs time for three drugs labeled A, B and
C. Drug A and B were given at identical doses while C was given at 1/100 the dose of A and B.

Which of the following statements is true.


a) half-life of A is equal to C and less than B
b) Vd of A is equal to C and greater than B
c) clearance of A is less than B and equal to C
d) Vd of A is equal to B greater than C
e) clearance of A is less than B and greater than C
Error! Filename not specified.
B. For a weak base, pH = pKa + (log (B/BH+))
So on pH = 5 side, base levels should be equal to protonated base. 
on pH 3 side, -2 = log (B/HB+), or 10^-2 = B/HB+, or a ratio of unionized base to protonated,
ionized base of 1:100. Levels of unionized base would be equal across lipid membrane, so that
means there is 100 times the protonated base on the pH = 3 side than there is on the pH = 5
side. Since there is 5 times the volume on the pH=3 side, that means there is 5 times the mass.
5*100 = 500.
Two compartments, pH = 5 and pH =3, are separated by a lipid membrane. Into the pH =5
compartment is placed a weak base with pK = 5. The volume of the compartment with pH=3 is 5
times the compartment with pH =5. At equilibrium there will be:

a) the same concentration in each compartment


b) approximately 500 times the mass in the pH =3 compartment as in the pH equal to 5
c) approximately 500 times the concentration in the pH =3 compartment as in the pH equal to 5
d) approximately 100 times the concentration in the pH =5 compartment as in the pH equal to 3
e) approximately 50 times the concentration in the pH =3 compartment as in the pH equal to 5
E. 99%

2.5 = 4.5 + log (A-/HA)


-2 = log (A-/HA)
1/100 = A-/HA, so 99% is in HA form.
Think of ASAcid, which is well absorbed in the gastric mucosa.
A weak organic acid with a pK of 4.5. What percentage of the acid will be in the lipid soluble
form at stomach pH of 2.5?
a) 1%
b) 10%
c) 50%
d) 90%
e) 99%
C. 130 ug

Vd = dose/Cp(0)
39000 mL = (0.3dose) / (0.1ug/100mL)
39 ug /0.3 = dose
Dose = 130 ug
A 70 kg lean male is taking drug A orally. The drug is 30% absorbed within minutes after oral
administration. The drug distributes evenly in all body water compartments without being
sequestered in any compartments and without binding to plasma proteins. The Vd of the drug is
39 liters. What loading dose of the drug should be administered to achieve a plasma
concentration of 0.1 μg/100 ml?
a) 700 μg
b) 390 μg
c) 130 μg
d) 39 μg
e) 13 μg
D. Greater surface area in small intestine than other areas of gut.
Most weak acids as well as weak bases are absorbed primarily from the small intestine after
oral administration because:
a) both types of drug are more ionized in the small intestine
b) both types of drug are less ionized in the small intestine
c) the blood flow is greater in the small intestine than in the other parts of the gut
d) the surface area of the small intestine is greater than most other parts of the gut
e) the small intestine has nonspecific carriers for most drugs
B
Twenty (20) mg of a non-polar drug whose molecular weight is 250 is administered
intravenously as a single bolus dose to a 50 kg woman. The extracellular volume is 9.5 liters,
interstitial volume is 7.5 liters and intracellular volume is 22 liters. Taking into account the
properties of the molecule, the most likely explanation for the results presented in the figure is:

a) the drug is unable to pass across capillaries and is entirely contained within the plasma
volume.
b) the drug is highly bound to plasma proteins and is sequestered in the plasma volume
c) the drug is rapidly distributed throughout the total body water
d) the drug is contained within the extracellular water
e) the drug is sequester in fat deposits.
Error! Filename not specified.
B. 0.4 mg /min

Css = (F * dosing rate) / Cl


Css = 10mg/L
F = 50% = 0.5
Cl = 20ml/min = 0.020L/min
Css/(F/Cl) = dosing rate
dosing rate = 10mg/L / (0.5/0.2) = 0.4 mg/min
You are administering a drug orally that is 100% absorbed but 50% of the drug undergoes first
passage metabolism. The desired therapeutic plasma concentration is 10 mg/liter. Clearance is
20ml/min and Vd is 40 liters. What dosing regimen should you give?
a) 0.2 mg/min
b) 0.4 mg/min
c) 0.8 mg/min
d) 2 mg/min
e) 4 mg/min
B.
Which of the following allows for transport of large macromolecules.
a) passive diffusion
b) pinocytosis
c) active transport
d) facilitated diffusion
e) passage through water channels
C. Rate of elimination is dependent on plasma concentration, there is a half life. Unlike zero
order kinetics in which drugs are eliminated by carrier-mediated mechs that can become
saturated, there is no half life.
If the plasma concentration of a drug declines by first-order kinetics:
a) the rate of elimination is proportional to the rate of administration at all times
b) the half-life of the drug depends on the plasma concentration
c) the rate of elimination is dependent on the plasma concentration
d) the rate of elimination is a constant.
e) none of the above
C. 1 mg/hr

Dm = (Css Cl 1 hr)/F
F=1
Cp(0) = 100 ng/mL
t(1/2) = 7 hours
dose = 1.0 mg
Css = 1ug/mL
First, you need to find Cl
Cl = k(e) * Vd
k(e) = 0.7/t(1/2) = 0.1
Vd = dose/Cp(0)
Cp(0) from the graph is 100 ng/ml = 100e-6 mg/ml
Vd = 1 mg / (100e-6 mg/ml) = 10000 mL = 10 L
Back to Cl = (0.1 * 10 L) = 1 L/hr

Dm = (1 ug /mL)(1000mL/hr)1hr = 1000 ug = 1 mg/hr


Drug D follows first order elimination as shown in the figure below. The drug will be
administered I.V. The drug in the figure below was administered at a dose of 1.0 mg and the
half-life of the drug is 7 hour. What maintenance dosing regimen must be administered to
maintain a therapeutic concentration of 1 μg/ml?

a) 10 μg/hr
b) 100 μg/hr
c) 1 mg/hr
d) 10 mg/hr
e) 100 mg/hr
Error! Filename not specified.
A. Concentration of plasma = concentration interstitial space
A patient is administered a drug that does not cross membranes, and does not bind to serum
proteins. The patient's hematocrit is 0.4, blood volume is 5 liters, and interstitial volume is 12
liters. The amount of drug in the interstitial volume is 12 mg. What is the plasma concentration
of the drug?
a) 1mg/liter
b) 1.2 mg/liter
c) 4 mg/liter
d) 2.2 mg/liter
e) 8 mg/liter
C. 3mg. Plasma volume = blood volume * (1-hematocrit) = 3 L. Drug concentration of plasma =
drug concentration of interstitial space = 1mg/L, so 3 mg
A patient is administered a drug that does not cross membranes, and does not bind to serum
proteins. The patient's hematocrit is 0.4, blood volume is 5 liters, and interstitial volume is 12
liters. The amount of drug in the interstitial volume is 12 mg. What is the mass amount of drug in
the plasma?

a) 1 mg
b) 1.2 mg
c) 3 mg
d) 4 mg
e) 6 mg
C
Drug A is 60% bound to albumin and the plasma concentration of drug A that is not bound to
albumin is 12 mg/liter. The patient is treated with a second drug that competes for albumin
binding. After addition of the second drug only 20 % of the drug A is bound. What is the new
free concentration of drug A immediately after addition of the second drug (i.e. before any of
drug A is eliminated)?
a) 1.4 mg/liter
b) 14 mg/liter
c) 24 mg/liter
d) 1.2 mg/liter
e) 12 mg/liter
C
Your patient is being administered a drug orally and you find that there are large fluctuations
between administrations. You do not want to change the therapeutic concentration. You should:

a) increase the loading dose


b) increase the number of times a day the drug is given and increase the dose
c) increase the number of times a day the drug is given and decrease the dose
d) decrease the number of times a day the drug is given and decrease the dose
e) decrease the number of times a day the drug is given and increase the dose
B. 29 mg

Dm = (Css Cl Tm )/F
F=1
Css = 1mg/L
Cl = 4.8 L/hr
Tm = 6 hours
Dm = 28.8 mg
Mr. Jones is admitted to General Hospital with pneumonia due to gram-negative bacteria. The
antibiotic tobramycin is ordered. The clearance (cl) and Vd of tobramycin for Mr. Jones are 4.8
liters/hr and 40 liters, respectively. What approximate maintenance dose should be
administered intravenously every 6 hours to eventually obtain average steady- state plasma
concentrations of 1mg/L?
a) 19 mg
b) 29 mg
c) 115 mg
d) 160 mg
e) 230 mg
C. chronic beta-blocker treatment caused supersensitivity of cardiac beta receptors
A patient you have been treating for hypertension with propranolol, suddenly stops taking his
medication and develops heart palpitations and rhythm irregularities. What receptor mechanism
might explain his current problem?
a. chronic beta-blockers cause down regulation of cardiac beta-receptors
b. beta-blockers are still in his bloodstream exerting an effect one day after stopping
c. chronic beta-blocker treatment caused supersensitivity of cardiac beta receptors
d. he should have been on another type of antihypertensive medication
e. over time, beta-blocker treatment caused desensitivity of beta-receptors
...
Which of the following statements best describes drugs classified as partial agonists?
a. partial agonists are prodrugs that split into full agonists in the body
b. full agonists are more potent that partial agonists
c. partial agonists have less efficacy than full agonists
d. partial agonists are also called negative antagonists
e. competitive antagonists used at a low concentration act like partial
agonists
...
Which of the following statements is correct concerning the therapeutic index (TI)?
a. TI is a measure of unit cost of drug per administration
b. penicillin has a smaller TI than warfarin
c. TI is the ratio of LD50 toxic effect/ED50 beneficial effect
d. the smaller the value of the TI, the safer the drug
e. LD50 values an equal to the TI for most drugs
...
Referring to the figure, which statement is correct?
a. A could be a full agonist, B a partial agonist and C an antagonist
b. A has more efficacy than B or C
c. C is the most potent drug
d. Curves B and C could be drug A plus increasing doses of competitive antagonist
e. maximal efficacy and the ED50 values of A, B, and C are the same
Error! Filename not specified.
...
Referring to the figure, which statement is correct?
a. A could be full agonist, B a partial agonist and C an antagonist
b. A has more efficacy than B or C
c. the rank order of potency is A>B>C
d. curves B and C could be drug A plus increasing doses of a noncompetitive
antagonist
e. maximal effect and the ED50 values of A,B and C are different
Error! Filename not specified.
...
An advertisement in a local newspaper seeks to enroll 200 patients with arthritis in a
medical study that would be the first time this new medicine would be tested in persons
with this disease. The study would therefore be classified as which type of trial?
a. phase 1 clinical study
b. phase II clinical study
c. phase III clinical study
d. phase IV clinical study
e. phase o preclinical study
...
A 25 year-old woman presents with muscle weakness, drooping eyelids and double
vision that she states worsened over the past 3-4 months. These symptoms are rapidly
reversed when she is given i.v. edrophonium for diagnostic purposes. Which of the
following agents might best treat her disease.
a. bethanechol
b. physostigmine
c. pilocarpine
d. pyridostigmine
e. tacrine
...
A 48 year old man is diagnosed with Sjogren's Syndrome, which is characterized by
chronic dry mouth due to undersecretion of saliva (xerostomia). Which of the following
agents might best treat this condition?
a. atropine
b. physostigmine
c. pilocarpine
d. pyridostigmine
e. donepezil
...
A 55 year old man suffers a myocardial infarction (MI or heart attack) and is stabilized in
the hospital. The next day his heart rate is less than 35 beats per min. Which of the
following agents might be administered to treat him?
a. atropine
b. sucinylcholine
c. tubocurarine
d. pilocarpine
e. pyridostigmine
...
A 24 year old medical student is taking his first spring break cruise and is worried about
getting sea sick on the ship. He asks his physician what medicine he can take so he does
not spend the entire cruise hanging over the railing. What agent that conveniently comes
in a skin patch formulation would his doctor recommend?
a. atropine
b. tolterodine
c. tubocurarine
d. ipratropium
e. scopolamine
...
A 9-year old boy is rushed to the Emergency Department wheezing, with labored
breathing and falling blood pressure. His mother tells the physician that he was stung by a
bee about 30 minutes ago. Which agent is given immediately to counteract the boy's
condition?
a. dobutamine
b. epinephrine
c. clonidine
d. ephedrine
e. amphetamine
...
A 16-year old athlete suffers from asthma after running for a touchdown. He is promptly
treated by using an inhaler and recovers in 10 minutes. Which one of the following agents
is most likely to be the active ingredient in the inhaler?
a. albuterol
b. epinephrine
c. clonidine
d. dobutamine
e. ephedrine
...
A 60 year old man is admitted to the hospital with an acute myocardial infarction (MI)
which leads to congestive heart failure. Which of the following agents known to
increase survival in patients, is most likely administered to the patient?
a. phenoxybenzamine
b. pindolol
c. carvedilol
d. dobutamine
e. dopamine
...
Which of the following drugs will decrease heart rate, cardiac contractility, and renin
secretion?
a. physotigmine
b. phentolamine
c. dobutamine
d. atropine
e. metoprolol
...
An elderly woman is found to exhibit elevated intraocular pressure, and open angle
glaucoma is diagnosed. Her physician prescribes pilocarpine every 6 hours. The
anticipated effect of pilocarpine eye drops would be to
a. relax the ciliary muscle
b. improve accommodation
c. relax the sphincter muscle of the iris
d. inhibit the production of aqueous humor by the ciliary epithelium
e. contract the longitudinal ciliary muscle and pull on the trabecular network to
relieve pressure
...
In a log dose-response plot, drug efficacy is determined by the maximal height of the
measured response on the effect axis, whereas drug potency is determined by:
a. number of animals exhibiting an all or none response
b. signal transduction pathway
c. formula, including the affinity of the drug and the number of drug receptors
d. position of the curve on the log-dose axis
e. steepness of the dose response curve
...
Which of the following drug interaction mechanisms is most likely to lead to sustained
elevations of plasma drug concentrations and drug toxicity?
a. induction of cytochrome CYP2C19
b. inhibition of cytochrome CYP3A4
c. displacement of a drug from plasma albumin binding sites
d. inhibition of the P-glycoprotein carrier protein
e. acceleration of gastric emptying by a "prokinetic" drug
...
Elderly persons have altered drug disposition because of
a. marked reduced absorption of many drugs
b. higher volumes of distribution of water soluble drugs
c. accelerated excretion of ionized drugs
d. increased permeability of the blood brain barrier
e. reduced capacity to oxidize drugs
...
If a test dose of edrophonium causes increased muscle weakness in a woman treated for
myasthenia gravis with pyridostigmine, it indicates that
a. the dose of edrophonium was too large
b. the dose of edrophonium was too small
c. the patient does not have myasthenia gravis
d. the dose of pyridostigmine should be increased
e. the dose of pyridostigmine should be reduced
...
Pyralidoxime improves muscle strength in persons exposed to toxic levels of an
organophosphate compound by
a. an action that leads to increased degradation of acetylcholine
b. binding to and activating nicotinic receptors
c. increasing the elimination of organophosphate compounds
d. increasing acetylcholine levels at cholinergic synapses
e. selectively blocking muscarinic receptors
...
Which effect is produced in the ciliary body after topical ocular administration of
clonidine or a derivative, apraclonidine?
a. increased inositol triphosphate (IP3)
b. increased cyclic AMP
c. decreased cyclic AMP
d. increased cyclic GMP
e. decreased cyclic GMP
...
Which drug is lease likely to slow recovery from hypoglycemia in a diabetic patient who
has taken an excessive dose of insulin?
a. metoprolol
b. propranolol
c. pindolol
d. labetalol
e. nadolol
...
For each patient described, select the most appropriate drug from the lettered sources
a. labetalol
b. phenoxybenzamine
c. propranolol
d. tamsulosin
e. phentolamine

23. A woman experiences pain and ischemia in her finger after accidentally injecting it with
an epinephrine auto injector that she carries for emergency treatment of allergic reactions

24. A man complains of urinary urgency, frequency and nocturia and is found to have benign
enlargement of the prostate gland

25. A woman with essential hypertension requires a drug that reduces both cardiac output
and peripheral resistance
26. A patient with episodic hypertension is found to have markedly elevated levels of
epinephrine and norepinephrine metabolites in his urine, and requires a long acting drug
to lower blood pressure before surgery
...
A patient suffering from depression and allergic to SSRI (selective serotonin reuptake
inhibitors) and tricyclic antidepressants was administered a monoamine oxidase inhibitor
(MAOI). You inform this patient that certain foods containing tyramine could now be
absorbed and could cause a hypertensive crisis. The primary site of action of tyramine is
a. ganglionic receptors
b. gut and liver catechol-o-methyltransferase
c. postganglionic sympathetic nerve terminals
d. vascular smooth muscle cell receptors
e. preganglionic sympathetic nerve terminals
...
Although it does not act at any histamine receptors, epinephrine reverses many effects of
histamine. Epinephrine is a
a. competitive inhibitor of histamine
b. noncompetitive antagonist of histamine
c. physiological antagonist of histamine
d. chemical antagonist of histamine
e. metabolic inhibitor of histamine
...
The heart rate response to the infusion of a moderate dose of phenylephrine in a
conscious patient is not blocked by
a. atropine
b. trimethaphan
c. phenoxybenzamine
d. reserpine
e. scopolamine
...
A new drug was administered to a group of normal volunteers. Intravenous bolus doses
produced the changes in blood pressure and heart rate shown in the graph below. The
most probable receptor affinities of this new drug are:

a. alpha1, alpha2 and beta1


b. alpha1, and alpha2 only
c. beta1 and beta2 only
d. muscarinic M3 only
e. nicotinic Nn only
Error! Filename not specified.
...
A 70-year old woman fell two years ago and broke her hip. Now she is being treated for a
blood pressure of 170/100 mmHg. When treating hypertension chronically, orthostatic
(postural) hypotension is greatest with
a. clonidine
b. guanadrel
c. reserpine
d. propranolol
e. nadolol
...
A patient has angina of effort (exertional angina) and chronic obstructive pulmonary
disease COPD. Which of the following drugs would you likely avoid in treating this
patient.
a. ipratropium
b. albuterol
c. terbutaline
d. propranolol
e. metoprolol
...
Drug Y has been shown to inhibit insulin secretion, increase blood glucose levels via
liver glycogenolysis and increases free fatty acids. Following the administration of
phentolamine, Drug Y increases insulin secretion. Drug Y is most likely
a. isoproterenol
b. terbutaline
c. phenylephrine
d. dobutamine
e. epinephrine
...
The figure above shows the blood pressure and heart rate response to epinephrine before
(1) and after (2) the administration of propranolol. A drug that would block the decrease
in heart rate following the second administration of epinephrine would be:
a. prazosin
b. reserpine
c. neostigmine
d. atropine
e. metaproterenol
Error! Filename not specified.
...
Carvedilol produces a greater decrease in blood pressure than many other beta adrenergic
receptor antagonists. The most likely explanation is that carvedilol:
a. decreases the release of norepinephrine during sympathetic neurotransmission
b. possesses substantial alpha1-adrenergic receptor blocking properties
c. decreases centrally mediated sympathetic neuronal outflow
d. release nitric oxide from vascular endothelial cells
e. possesses substantial beta2 agonism
...
Many drugs have pharmacological effects due to central actions after they cross the blood
brain barrier. Which of the following drugs is most likely to enter the central nervous
system.
a. physostigmine
b. tubocurarine
c. edrophonium
d. guanethidine
e. succinylcholine
...
37. Which drug is most potent?
a. drug A
b. drug B
c. drug C
d. drug D
e. not enough information to determine
38. When used at a maximally effective therapeutic dose (i.e. ED99) which drug will produce
the highest incidence of toxicity.
a. drug A
b. drug B
c. drug C
d. drug D
e. not enough information to determine
Error! Filename not specified.
...
An anesthetized patient or experimental animal is premedicated with atropine and
pindolol. An intravenous dose of norepinephrine would then produce which of the
following responses?

a. increase in arterial pressure, relaxation of the smooth muscle of the GI tract, and
contraction of the radial muscle of the iris
b. increase in arterial blood pressure, increase in heart rate and relaxation of the
radial muscle of the iris
c. decrease in blood pressure and relaxation of the smooth muscle of the GI tract
d. no change in heart rate, no change in blood pressure and no change in the tone of
the radial muscle of the iris
e. decrease in arterial blood pressure and no change in heart rate
...
Susceptibility to cholinesterases: -
Pharmacological Actions
Cardiovascular: +/-
GastroIntestinal: +++
Urinary
Bladder +++
Eye (topical) ++
Antagonism by Atropine +++
Nicotinic -

The above table depicts the pharmacological profile of a drug (DRUG A) that acts on the
cholinergic system. From the profile you would suppose the drug is most likely:
(+ = degree of activity)
a. pilocarpine
b. acetylcholine
c. methacholine
d. bethanechol
e. neostigmine
...
A drug (Drug 1) was given as an IV bolus to a subject while blood pressure and
heart rate were recorded as shown on the left side of the graph below. After recovery from the
effects of Drug 1, a long-acting dose of Drug 2 was given. After the recorder was turned back
on,
Drug 1 was repeated with the results shown on the right side of the graph.

Identify Drug 1 from the following list


a. phenylephrine
b. norepinephrine
c. angiotensin
d. isoproterenol
e. prazosin
Error! Filename not specified.
...
A drug (Drug 1) was given as an IV bolus to a subject while blood pressure and
heart rate were recorded as shown on the left side of the graph below. After recovery from the
effects of Drug 1, a long-acting dose of Drug 2 was given. After the recorder was turned back
on,
Drug 1 was repeated with the results shown on the right side of the graph.

Identify Drug 2 from the following list


a. propranolol
b. mecamylamine
c. terazosin
d. guanethidine
e. isoproterenol
Error! Filename not specified.
...
Methylation of drugs via a methyl transferase and the substrate S-adenosylmethionine is a
relatively minor pathway in drug metabolism. On the other hand it is important in the
metabolism of catecholamines. It is an example of which of the following

a. phase 1 biotransformation reaction


b. CYP450 biotransformation reaction
c. phase 2 biotransformation reaction
d. pathway in the synthesis of catecholamines
e. non-microsomal; non-CYP450 biotransformation reaction
...
The most important quantitative phase 2 biotransformation reaction is
a. acetylation
b. glucuronidation
c. glutathione conjugation
d. sulfate conjugation
e. carbonyl reduction
...
During a laboratory demonstration to depict the complexity of neurotransmission in
autonomic ganglia, Professor PharmPhys sets up an anesthetized mammalian preparation in
which he is recording postsynaptic events following the electrical stimulation of preganglionic
sympathetic nerves. This demonstrates a complex action potential consisting of a fast EPSP
followed by a slow IPSP, followed by a slow EPSP and finally by a very late and slow EPSP
(see
figure below).

In Professor PharmPhys's demonstration, a mediator of the late slow EPSP could be:
a. dopamine
b. acetylcholine
c. neuropeptide Y
d. an autacoid (eg. angiotensin)
e. nicotine
Error! Filename not specified.
...
During a laboratory demonstration to depict the complexity of neurotransmission in
autonomic ganglia, Professor PharmPhys sets up an anesthetized mammalian preparation in
which he is recording postsynaptic events following the electrical stimulation of preganglionic
sympathetic nerves. This demonstrates a complex action potential consisting of a fast EPSP
followed by a slow IPSP, followed by a slow EPSP and finally by a very late and slow EPSP
(see
figure below).

The principle receptor mediating the fast EPSP is:


a. M2 muscarinic cholinergic
b. M1 muscarinic cholinergic
c. nicotinic cholinergic
d. M3 muscarinic cholinergic
e. dopaminergic or alpha2 adrenergic
Error! Filename not specified.
...
The above figure depicts a signaling pathway for some types of drugs and hormones. If
Drug A is an agonist that acts on a receptor that works through this signaling pathway
which of the following could block the effect of Drug A
a. pinolol
b. methyldopa
c. trimethaphan
d. propranolol
e. terazosin
Error! Filename not specified.
...
The graph above shows the results of an experiment in which the overflow of
norepinephrine caused by nerve stimulation of an isolated vascular smooth muscle was
measured
in the absence (control) or presence of drug X or drug Y.

Drug X could be
a. cocaine
b. acetylcholine
c. propranolol
d. guanethidine
e. dobutamine

Drug Y could be
a. prazosin
b. phenoxybenzamine
c. clonidine
d. terbutaline
e. phenylephrine
Error! Filename not specified.
...
A patient suffering from cardiogenic shock is treated with drug X that increases blood
flow through the mesenteric and renal vascular beds, activates alpha1 adrenergic receptors in
several other vascular beds and directly and indirectly stimulates beta1 adrenergic receptors
in the heart. The effect of drug X on the mesenteric and renal vascular beds can be
blocked by a drug that antagonizes:
a. beta2-adrenergic receptors
b. muscarinic-cholinergic receptors
c. dopamine receptors
d. the release of epinephrine from the adrenal medulla
e. alpha-adrenergic receptors
...
A patient suffering from nasal congestion is being treated with drug X. Drug X produces
its pharmacological effect by activating a receptor linked via a G-protein to the opening
of membrane Ca2+ channels and to activation of phospholipase C with a resultant increase
in two second messengers. A second messenger formed following treatment with drug X
is:

a. cyclic AMP
b. cyclic GMP
c. nitric oxide
d. inositol triphosphate
e. arachidonic acid
...
During the catalytic cycle of the cytochrome P450-dependent monooxygenase system the
ferric (Fe+3) hemocytochrome P-450 substrate complex is reduced by a single electron to
the ferrous (Fe+2) hemocytochrome complex. The electron is provided by:
a. NADPH
b. NADP+
c. aldehyde dehydrogenase
d. molecular oxygen
e. GSH-S-transferase
...
Intravenous administration of Drug X to an anesthetized or unanesthetized mammal or
human produces a pressor response (increase in blood pressure). After administration of
Drug Y, Drug X produces no change in blood pressure. Which of the following pairs of
drugs would NOT produce the above sequence of events? (Drug X to Drug Y)

a. phenlyephrine phentolamine
b. ephedrine prazosin
c. isoproterenol atropine
d. norepinephrine phenoxybenzamine
e. amphetamine terazosin
...
Beta adrenergic blocking agents are very useful drugs and can be used to treat numerous
clinical conditions. Because of this there are many beta blockers on the market. In which
way does metoprolol differ from propranolol?

a. metoprolol is used for the management of hypertension


b. metoprolol has some selectivity for beta2 adrenoceptors
c. metoprolol is less likely to decrease cardiac output
d. metoprolol is less likely to precipitate bronchoconstriction
e. metoprolol inhibits release of renin from the kidney
...
All drugs produce untoward effects. If a drug has the following characteristics:
seen in only genetically abnormal subjects; the incidence affects few drugs and
prior exposure is unnecessary
it would be called an:
a. allergic response
b. idiosyncratic response
c. toxic response
d. tachyphylaxis
e. drug interaction
...
The following diagram depicts the effect of various drugs on blood pressure in an anesthetized
mammal before and after pretreatment with another drug:
(ACh - acetylcholine; Epi - epinephrine; ISO - isoproterenol; NE - norepinephrine)

Which pretreatment was used:


a. atropine
b. cocaine
c. prazosin
d. propranolol
e. phenoxybenzamine
Error! Filename not specified.
...
The following diagram depicts the effect of various drugs on blood pressure on an anesthetized
mammal before and after pretreatment with another drug.
(Ach - acetylcholine; NO - nitric oxide; EpL - epinephrine; ISO - isoproterenol; NE -
norepinephrine)

Which pretreatment was used?


a. propranolol
b. prazosin
c. atropine
d. phenoxybenzamine
e. ephedrine
Error! Filename not specified.
...
A patient requires an infusion of procainamide. Its half-life is 2 hours. The infusion is
begun at 9 AM. At 1 PM of the same day a blood sample is taken, and the drug plasma
concentration is found to be 3 mg/L. What is the steady state drug concentration after 48
hours of infusion?
a. 3 mg/L
b. 4 mg/L
c. 6 mg/L
d. 9.9 mg/L
e. 15 mg/L
...
A normal volunteer will receive a new drug in a phase I clinical trial. The clearance and
volume of distribution of the drug in this subject are 1.4 L/h and 80 L, respectively. The
half-life of the drug in this subject will be approximately
a. 83 hours
b. 77 hours
c. 58 hours
d. 40 hours
e. 2 days
...
Gentamicin is often given in intermittent IV bolus doses of 100 mg three times a day to
achieve steady state plasma concentrations of about 5 mg/L. Gentamicin's clearance
(normally 5.4 L/h/70 kg) is almost entirely by glomerular filtration. Your patient,
however, is found to have a clearance one-third of normal. Your initial dosage regimen
for this patient would probably be
a. 20 mg three times a day
b. 33 mg three times a day
c. 72 mg three times a day
d. 100 mg twice a day
e. 150 mg twice a day
...
You determine that the hematocrit of your patient is 0.4. The blood concentration is
176microg/L. The drug is unionized. The plasma concentration is:
a. 176microg/L
b. 50microg/L
c. 200microg/L
d. 266microg/L
e. 300microg/L
...
Quinine, a weak base is ionized in the blood, (pH =7.4)). The pKa of the drug is 8.4:
The per cent of drug in the ionized form is approximately.
a. 10%
b. 50%
c. 80%
d. 90%
e. 99%
...
If 100 mg of a drug is given orally every 8 hours to a 70 kg male patient. The drug is
eliminated according to zero order kinetics. Steady- state concentration of the drug will
be achieved in:
a. 14-18 hours
b. 20-24 hours
c. 1 day
d. 1 to 2 days
e. Never
...
You decide to treat a 60kg man by prescribing drug A to be given orally. The drug is
50% absorbed in the small intestine and does not undergo first pass biotransformation.
The clearance of the drug is 1L/kg/hr and the half-life is 7 hours. The desired therapeutic
concentration is 4ug/liter and Vd is 10L/kg. Calculate the loading dose:
a. 1.2mg
b. 2.4mg
c. 3.2mg
d. 4.8mg
e. 6.4mg
...
You administer a bolus dose of a drug I.V. The dose is
110 mg and blood samples were analyzed at intervals as shown in the graph above.

Calculate the half-life of the drug.


a. 40-60minutes
b. 60-80 minutes
c. 80-100 minutes
d. 120-140 minutes
e. 160-180 minutes
Error! Filename not specified.
...
You administer a bolus dose of a drug I.V. The dose is
110 mg and blood samples were analyzed at intervals as shown in the graph above.

Calculate the volume of distribution


a. 1ml
b. 2ml
c. 1 liter
d. 2 liter
e. 20 liter
Error! Filename not specified.
...
The bioavailability of drug A is equal to drug B. When each drug is injected I.V. and
blood samples are taken; the drug concentration curves seen below are found. Which
statement concerning drugs A and B is true?

a. The AUC(A) = AUC(B)


b. The AUC(A) is less than AUC(B)
c. The AUC(A) is more than AUC(B)
d. Half-life of A is equal to the drug B's half-life
e. Ke of A is greater than drug B's Ke
Error! Filename not specified.
...
A patient with congestive heart failure is receiving digoxin at a daily dose of 0.25 mg
orally. A digoxin concentration in plasma is reported at 1.2 ng/ml after steady-state
conditions have been reached (therapeutic range for plasma digoxin is 0.8-2.0 ng/ml).
You are asked to evaluate this patient because of declining renal function, and it is
estimated that the creatinine clearance is now 50% of normal. 
What would you
recommend with regard to the daily dose of digoxin?
a. Reduce the daily dose by 25%
b. Stop dioxin
c. Reduce the daily dose by 75%
d. Continue the same daily dose
e. Give 0.25 mg every 72 hours
...
You are testing two drugs, drug A and drug B. You find that drug A has a larger
apparent volume of distribution (Vd) than B. The half-lives of both drugs are equal. The
different volumes of distribution is likely due to:
a. A higher degree of plasma protein binding of drug A
b. A larger first-pass effect of drug A when given orally
c. A higher renal and hepatic clearance of drug A
d. A shorter plasma elimination half-life of drug A
e. A higher tissue affinity relative to plasma of drug A
...
You are administering a drug and only its ionized form is effective. The plasma
concentration of the drug (effective and non effective) is 1.1mg/L. The pH of the plasma
is 7.4 and the pKa of the drug is 6.4. Calculate the amount of effective drug in the
plasma.
a. 1μg/L
b. 0.01mg/L
c. 0.1mg/L
d. 1mg/L
e. 0.1mg/ml
...
You have prescribed a drug given orally. The drug is 70% absorbed and does not
undergo first pass biotransformation. The therapeutic concentration is 1.0mg/L. The
half-life of the drug is 4 hours and the volume of distribution is 40L. What dosing
regimen should you administer?
a. 1mg/hour
b. 2mg/hour
c. 5mg/hour
d. 8mg/hour
e. 10mg/hour
...
Drug A has a half-life equal to drug B. Both are administered with identical dosing
regimens (i.e. dose/dosing intervals are equal). Both drugs have identical volumes of
distribution. Drug A is 40% absorbed and does not undergo first pass biotransformation.
Drug B is 60% absorbed and 50% of the drug undergoes first pass biotransformation.

Which of the following is true:


a. The steady state plasma concentration of A is greater than B
b. The steady state plasma concentration of A is less than B
c. The bioavailability of A is less than B
d. The clearance of A is less than B
e. The clearance of A is greater than B
B
A 39-year-old female is given a muscle relaxant prior to endotracheal intubation. Minutes later
the patient is noted to have a sudden onset of hyperthermia, tachycardia, sweating, cyanosis,
tachypnea and muscle rigidity. The most likely muscle relaxant causing these symptoms is:

a) vecuronium
b) succinylcholine
c) atracurium
d) miracurium
e) d-tubocurarine
D
A 51-year-old male has been treated over the last 3 months for hypertension with a
medication that works by inhibiting the transport of norepinephrine from the neuronal cytoplasm
into synaptic vesicles. On a check-up visit to his primary care physician, the patient notes that
he has been feeling "depressed and sluggish" over the last week. The
patient was most likely receiving:
a) guanadrel
b) clonidine
c) methyldopa
d) reserpine
e) propranolol
E. Cimetidine is an important inhibitor of CYP1A2, as is grapefruit juice
A 46-year-old female with asthma has been stabilized on theophylline, a drug
metabolized by CYP1A2. She has been treated for 6 months without any complaints of adverse
effects. One week after the patient went to the pharmacy to purchase an over-the-counter
(OTC) medication to help relieve her "heartburn" symptoms she began experiencing
palpitations, tremors and nausea (side effects of theophylline). What was the most likely OTC
medication?

a) chlorpheniramine
b) atropine
c) ethanol
d) aspirin
e) cimetidine
C. Beta blockers are used to prevent cardiovascular effects seen in thyrotoxicosis
A 43-year-old female with thyroid dysfunction is currently thought to be in thyroid storm
(thyrotoxicosis). She is given a drug to prevent cardiovascular effects seen in thyrotoxicosis.
This drug was most likely:

a) clonidine
b) prazosin
c) propranolol
d) methyldopa
e) atropine
C. Histamine, because nothing else makes sense.
In experimental studies, when anesthetized subjects are treated intravenously with drug X, they
exhibit a marked decrease in blood pressure and a large reflex increase in heart
rate. Pretreatment of the subjects with either atropine or propranolol fails to block the decrease
in blood pressure. Drug X is most likely to be which of the following agents?

a) acetylcholine
b) epinephrine
c) histamine
d) isoproterenol
e) succinylcholine
B. Clonidine
The accompanying table shows cardiovascular changes induced by drug treatment in
experimental subjects. Which of the following drugs is most likely to cause changes shown in
Profile 4?

a) amphetamine
b) clonidine
c) epinephrine
d) isoproterenol
e) tyramine
Error! Filename not specified.
C. Epinephrine
Which of the following drugs would most likely cause the effects observed in Profile 2:

a) amphetamine
b) clonidine
c) epinephrine
d) isoproterenol
e) tyramine
Error! Filename not specified.
D. Isoproterenol
Which of the following drugs would most likely cause the effects observed in Profile 3:

a) amphetamine
b) clonidine
c) epinephrine
d) isoproterenol
e) tyramine
Error! Filename not specified.
D. Pilocarpine
A 71-year-old man with acute angle-closure (narrow angle) glaucoma is brought into the
emergency department because he is experiencing extreme pain in his left eye, is seeing halos
around lights and has an intensely red left eye and a steamy-appearing cornea. Which of the
following drugs would be considered appropriate medical treatment for this disorder?

a) atropine
b) edrophonium
c) timolol
d) pilocarpine
e) carvedilol
D. Succinylcholine is a depolarizing agent, initial stage of overactivation.
A 42-year-old man requires emergency tracheal intubation and is given succinylcholine. Which
of the following best described the molecular action of succinylcholine as a neuromuscular
blocker in this patient?

a) acetylcholinesterase inhibitor
b) muscarinic receptor agonist
c) muscarinic receptor antagonist
d) nicotinic receptor agonist
e) nicotinic receptor antagonist
E. Propanolol
In experimental studies when anesthetized subjects are treated intravenously with an agonist
(drug X), they have an increase in their blood glucose level and in the force of ventricular
contraction and a slight decrease in peripheral resistance. Pretreatment of the subjects with an
antagonist (drug Y) almost completely blocks all of these effects. Drug Y is most likely to be
which of the following agents?

a) metoprolol
b) atropine
c) guanethidine
d) phentolamine
e) propranolol
E
In experimental studies when anesthetized subjects are treated intravenously with drug X, they
show a marked increase in both systolic and diastolic blood pressure and a
decrease in heart rate. However, if they are pretreated with prazosin they show a modest to
slight increase in the systolic blood pressure and a marked increase in heart rate. 

Drug X is most likely to be which of the following agents?

a) acetylcholine
b) epinephrine
c) histamine
d) isoproterenol
e) norepinephrine
...
The accompanying figure depicts nerves in the autonomic and somatic nervous system. Five
sites are labeled with the numbers 1 through 5. Muscarinic receptors are commonly associated
with which site?

a) Site 1
b) Site 2
c) Site 3
d) Site 4
e) Site 5
Error! Filename not specified.
D
Norepinephrine is the neurotransmitter commonly associated with which site?
a) Site 1
b) Site 2
c) Site 3
d) Site 4
e) Site 5
Error! Filename not specified.
B
The neurotransmitter associated with Site 5 is:
a) norepinephrine
b) acetylcholine
c) serotonin
d) neuropeptide Y
e) epinephrine
Error! Filename not specified.
B.

Tolterodine is an antimuscarinic used to treat urinary incontinence.


The therapeutic use of tolterodine is based on its ability to:

a) relax bronchial smooth muscle


b) relax urinary bladder smooth muscle
c) relax uterine smooth muscle
d) inhibit salivary secretions
e) relax ciliary smooth muscle
D
A 25-year-old woman presents with muscle weakness, drooping eyelids and double vision that
she states worsened over the past 3-4 months. These symptoms are usually reversed when she
is given i.v. edrophonium for diagnostic purposes. The medication that you prophylactically treat
this patient with has which of the following molecular
mechanism of action:

a) activation of muscarinic receptors


b) blockade of nicotinic receptors
c) activation of alpha1 adrenergic receptors
d) inhibition of acetylcholinesterase
e) inhibition of acetylcholine release
B. Alpha-1.

Mydriasis is controlled by alpha-1 receptors, and hypotension that results from blocking alpha-1
receptors would produce reflex tachycardia.
A patient comes into your office and upon examination and careful history you observe the
following effects. Orthostatic hypotension, miosis and tachycardia. The patient had been taking
medication but couldn't remember the name. You surmise this was a drug that acted on which
of the following receptors:

a) M2 receptors
b) alpha1 receptors
c) nicotinic cholinergic receptors
d) beta1 adrenergic receptors
e) alpha2 adrenergic receptors
D
The above diagram represents concentration effect curves to norepinephrine showing
increases in mean arterial blood pressure in an anesthetized individual. Curve X and Y
represent the effect of norepinephrine in the presence of Drug X or Drug Y. Drug X is
most likely acting at which of the following:
a) vesicular amine transporter
b) uptake 2
c) monoamine oxidase
d) uptake 1
e) catechol-o-methyltransferase
Error! Filename not specified.
E
The above diagram represents concentration effect curves to norepinephrine showing increases
in mean arterial blood pressure in an anesthetized individual. Curve X and Y represent the
effect of norepinephrine in the presence of Drug X or Drug Y.

Drug Y is most likely acting on which of the following receptors:

a) alpha2 adrenergic
b) nicotinic cholinergic
c) beta1 adrenergic
d) muscarinic cholinergic
e) alpha1 adrenergic
Error! Filename not specified.
B
As a busy third year student you are working in an outpatient clinic and find yourself treating an
interesting patient. This young woman is a violinist for the famous St. Louis
symphony. She tells you she gets frightfully nervous before each performance. Her symptoms
include muscle tremor, diarrhea, nervousness and palpitations. She asks for some medication
to help. The drug you choose most likely works on which of the
following receptors:

a) alpha1 adrenergic
b) beta1 and beta2 adrenergic
3) muscarinic cholinergic
d) nicotinic cholinergic
e) alpha2 adrenergic
22. C
23. B, atracurium is a nondepolarizing neuromuscular relaxant
24. E
25. C, Tamsulosin is an alpha-1 blocker used in the treatment of BPH
The above diagram depicts targets for drug actions (A-->E) located in nerves or innervated
organs. Match the following drugs with their primary site of action. (Letter can be used more
than once).

22. carvedilol
23. atracurium
24. tacrine
25. tamsulosin
Error! Filename not specified.
C. Terazosin, which is an alpha-1 blocker, used in the treatment of BPH. It leads to symptoms of
urinary hesitancy, frequent urination, dysuria (painful urination), increased risk of urinary tract
infections, and urinary retention.
A 70-year-old male patient comes into your office complaining of urinary tract symptoms. He
states that he has hesitancy in voiding, needs to void with great frequency and has a poor
stream when voiding. A drug that would be useful in helping these symptoms would be which of
the following:

a) tolterodine
b) propranolol
c) terazosin
d) phenylephrine
e) terbutaline
B. Phenylephrine, which activates alpha-1 receptors, which are linked to PLC and IP3, DAG and
Ca2+
A patient suffering from nasal congestion is being treated with drug Y. Drug Y produces its
pharmacological effect by activating a receptor that is linked via a Gprotein to the opening of
membrane Ca2+ channels and to the activation of phospholipase C with a resultant increase in
two additional second messengers. Drug Y is most likely:

a) edrophonium
b) phenylephrine
c) phentolamine
d) pindolol
e) pancuronium
D
The above graph depicts the hydrolysis and % inhibition of succinylcholine in two
patients (patients X and Y). Patient Y is one that has which of the following:
a) normal pseudocholinesterase
b) polymorphism of CYP3A4
c) polymorphism of N-acetylation
d) atypical pseudocholinesterase
e) polymorphism of glucuronal transferase
Error! Filename not specified.
C
According to the above table, C refers to a:

a) toxic response
b) idiosyncratic response
c) allergic response
d) placebo response
e) supersensitivity response
Error! Filename not specified.
E. CYP450 reactions include dehalogenation among other oxidation reactions. Phase II are
conjugation, and cyp-independent include the others
Cytochrome P450's (CYP 450) are an important family of enzymes involved in the
biotransformation of xenobiotics or drugs. Which of the following is a CYP450 reaction:

a) amine oxidation
b) alcohol dehydrogenation
c) acetylation
d) methylation
e) dehalogenation
C. Muscarinic, probably ipratropium
When applied to the airway of an asthmatic individual, drug X causes bronchodilation. The
effect is not blocked by labetalol. Drug X does not normally have any central nervous system
effect and if given by mouth has a bioavailability of less than 30%.

Drug X is most likely acting on which type of receptor:


a) alpha 2
b) alpha 1
c) muscarinic
d) histaminergic
e) nicotinic
B
Which of the following is the least likely clinical use for a beta-adrenergic antagonist:
a) glaucoma
b) atony of the urinary bladder
c) hyperthyroidism
d) migraine headache
e) congestive heart failure
C
A farmer is brought into the Emergency Department after an accident with crop spraying
equipment, during which he was copiously sprayed with an insecticide. He is salivating
profusely and has difficulty in breathing. His pulse rate is 40 beats/min. The most appropriate
treatment is:

a) atropine alone
b) atropine plus neostigmine
c) atropine plus pralidoxime
d) atropine plus ipratropium
e) atropine plus physostigmine
D
Unfortunately you have a wisdom tooth that has to be removed. You go to your favorite dentist
and he gives you a local anesthetic prior to proceeding to extract the tooth. The
local anesthetic he gives you is procaine. He explains that epinephrine is also present in
the solution containing procaine. Epinephrine is present in the procaine solution in order
to:

a) stimulate local wound repair


b) promote hemostasis
c) enhance the interaction with neuronal membranes to depress nerve conduction
d) retard the systemic absorption of procaine
e) facilitate the distribution of procaine along nerves
B
In an anesthetized subject, electrical recording electrodes are placed on:
I Carotid sinus baroreceptor nerve fibers
II Splanchnic (sympathetic) nerve fibers (preganglionic)
III Inferior cardiac (sympathetic) nerve fibers (postganglionic)
IV Vagal (parasympathetic) nerve fibers

What changes (if any) in firing rates of the nerves would be expected to occur following the
administration of the last drug in each series of drugs listed below? Presume that sufficient time
for the actions of the premedicating agents has been allowed and then the last agent is given
intravenously.

Give one answer for each set of nerves (in order I-IV) from the choices below.
= increase of nerve activity; = decrease nerve activity; ↔ = no change in neural firing

Propranolol, and atropine; then phenylephrine


Error! Filename not specified.
B
In an anesthetized subject, electrical recording electrodes are placed on:
I Carotid sinus baroreceptor nerve fibers
II Splanchnic (sympathetic) nerve fibers (preganglionic)
III Inferior cardiac (sympathetic) nerve fibers (postganglionic)
IV Vagal (parasympathetic) nerve fibers

What changes (if any) in firing rates of the nerves would be expected to occur following
the administration of the last drug in each series of drugs listed below? Presume that sufficient
time for the actions of the premedicating agents has been allowed and then the last agent is
given intravenously.
Give one answer for each set of nerves (in order I-IV) from the choices below.
= increase of nerve activity; = decrease nerve activity; ↔ = no change in neural firing

Propranolol, atropine and reserpine; then phenylephrine


Error! Filename not specified.
E
In an anesthetized subject, electrical recording electrodes are placed on:
I Carotid sinus baroreceptor nerve fibers
II Splanchnic (sympathetic) nerve fibers (preganglionic)
III Inferior cardiac (sympathetic) nerve fibers (postganglionic)
IV Vagal (parasympathetic) nerve fibers
What changes (if any) in firing rates of the nerves would be expected to occur following
the administration of the last drug in each series of drugs listed below? Presume that sufficient
time for the actions of the premedicating agents has been allowed and then the last agent is
given intravenously.
Give one answer for each set of nerves (in order I-IV) from the choices below.
= increase of nerve activity; = decrease nerve activity; ↔ = no change in neural firing

Propranolol, atropine, reserpine and hexamethonium (or mecamylamine); then


phenylephrine
Error! Filename not specified.
D
In an anesthetized subject, electrical recording electrodes are placed on:
I Carotid sinus baroreceptor nerve fibers
II Splanchnic (sympathetic) nerve fibers (preganglionic)
III Inferior cardiac (sympathetic) nerve fibers (postganglionic)
IV Vagal (parasympathetic) nerve fibers

What changes (if any) in firing rates of the nerves would be expected to occur following
the administration of the last drug in each series of drugs listed below? Presume that sufficient
time for the actions of the premedicating agents has been allowed and then the last agent is
given intravenously.
Give one answer for each set of nerves (in order I-IV) from the choices below.
= increase of nerve activity; = decrease nerve activity; ↔ = no change in neural firing

Propranolol, atropine, reserpine, hexamethonium (or mecamylamine) and


phenoxybenzamine; then phenylephrine
Error! Filename not specified.
A
An anesthetized individual is premedicated with propranolol and atropine. An
intravenous dose of norepinephrine would then produce which of the following
responses:

a) increase in arterial pressure, relaxation of smooth muscle of the GI tract and


contraction of the radial muscle of the iris
b) increase in blood pressure, increase in heart rate and relaxation of the radial
muscle of the iris
c) decrease in blood pressure, relaxation of the smooth muscle of the GI tract
d) no change in heart rate, no change in blood pressure, no change in radial smooth
muscle tone
e) decrease in arterial blood pressure, no change in heart rate
C
A drug that has high potential for abuse, theoretically has no current accepted medical
treatment in the United States and a lack of accepted safety for use under medical supervision
is classified by the DEA as:

a) schedule V drug
b) schedule II drug
c) schedule I drug
d) schedule III drug
e) unscheduled drug
E
The drug act that established guidelines for the truthful labeling of drugs and set
guidelines for the safety of drugs and established the Food and Drug Administration
(FDA) was:
a) Kefauver-Harris Amendments of 1962
b) Orphan Drug Amendments of 1983
c) Expediated Drug Approval Act of 1992
d) Pure Food and Drug Act of 1906
e) Food, Drug and Cosmetic Act of 1938
C
You have a patient that suffers from angina pectoris, hypertension and chronic obstructive
pulmonary disease (COPD).You are concerned about the use of beta blockers in this patient.
However, because of drug allergies to other drugs, you decide
to use a beta blocker anyway. The best choice in such a patient would be:
a) propranolol
b) labetalol
c) metoprolol
d) pindolol
e) carvedilol
C. A beta blocker with an alpha blocker
The effects depicted on the graph above on blood pressure were recorded in a subject before (I)
and after (II) pretreatment with a drug or combination of drugs. Which pretreatment regimen was
used (select a single drug in preference to a combination if
possible).

a) trimethaphan
b) phentolamine + propranolol
c) carvedilol + phenoxybenzamine
d) metoprolol
e) atropine
Error! Filename not specified.
B
Concerning clinical trials, which phase involves numerous physicians and is designed to provide
information on adverse drug reactions following use in large numbers of patients:
a) phase 1
b) phase 4
c) phase 3
d) phase 2
e) phase 5
C
Patient X is asked to climb a set of stairs. After a few minutes he is unable to continue.
Treatment with pyridostigmine allows him to continue to climb the stairs without difficulty. A side
effect that may occur in patient X is:

a) mydriasis
b) tachycardia
c) bronchoconstriction
d) dry mouth
e) hypertension
A
You are testing drug combinations containing two drugs. Both drugs bind to serum proteins.
Given alone drug A binds with a Kd of 10-7 while given alone drug B binds with a Kd of 10-3.
You give a bolus dose of drug that contains 1mg of each drug. Assume that there is no
difference in the distribution of the drugs. Which of the following statements is true concerning
the amounts of drug A and B?

a) there will be more of drug A in the plasma


b) there will be more of drug B in the plasma
c) drug A and B in the plasma will be the same
d) there will be more free (unbound) drug A in the plasma
e) there will be the same amount of free A and B in the plasma
C
A patient has a hematocrit of 0.5. The blood concentration of a drug is 10ug/liter. The drug is
unionized. Calculate the concentration of the drug in the plasma.

a) 6ug/liter
b) 8ug/liter
c) 12ug/liter
d) 20ug/liter
e) 40ug/liter
D
A pharmaceutical company is testing the clearance (Cl) of two drugs, drug A and drug B. The
drugs are cleared by the liver. Drug A is cleared at the rate of liver perfusion, drug B at much
lower rate. Both drugs are 50% bound to albumin. If liver perfusion rate is increased, which of
the following is likely true concerning the clearance of drugs
A and B?

a) the Cl of Drug A and drug B will stay the same


b) the Cl of Drug A will increase and drug B will decrease
c) the Cl of Drug A and drug B will increase
d) the Cl of Drug A will increase and drug B stay the same
e) the Cl of Drug B will increase and drug A stay the same
E.

D(L) = (Css * Vd)/F


You administer a loading dose to a 70 kg man to attain a therapeutic concentration of 10
mg/liter. The half-life of the drug is 2 hours, and the volume of distribution is 1 liter/kg. What
loading dose should you administer?

a) 5mg
b) 10mg
c) 70mg
d) 100mg
e) 700mg
C
In the absence of other drugs, pindolol causes an increase in heart rate by activating beta
receptors. However, in the presence of potent beta stimulants, administration of pindolol results
in a dose-dependant, reversible decrease in heart rate. Therefore, pindolol is:

a) noncompetitive antagonist
b) irreversible antagonist
c) partial agonist
d) spare receptor agonist
e) none of the above
D
Aspirin is a weak organic acid with a pKa of 3.5. What percentage of a given dose will be in the
lipid-soluble form at a stomach pH of 2.5?

a) 1%
b) 10%
c) 50%
d) 90%
e) 99%
E
Drug X is a weak acid, pKa = 6.0. It is eliminated almost entirely by glomerular filtration. Under
which of the following conditions would drug x likely be eliminated
most rapidly?

a) urinary pH = 4.5
b) urinary pH = 5.0
c) urinary pH = 6.0
d) urinary pH = 7.0
e) urinary pH = 8.0
D
A drug when administered at 3mg i.v. The following data has been collected for the plasma
concentration as a function of time. Which of the following answers best
describes these kinetics?

a) two compartments with a first order elimination rate


b) two compartments with a zero order elimination rate
c) one compartment with a zero order elimination rate
d) one compartment with a first order elimination rate
e) none of the above
Error! Filename not specified.
B
Drugs A and B, have the same mechanism of action. Drug A at a dose of 5 mg produces the
same magnitude of effect as drug B in a dose of 500mg.

a) drug B is less efficacious than drug A


b) drug A is 100 times more potent that drug B
c) toxicity of drug A is less than that of drug B
d) drug A is a better drug if maximal efficacy is needed
e) drug A has a shorter duration of action
B

Five half lives needed to reach steady state concentration after a dosing regimen
Drug A has a drug half-life 80% of Drug B. Both drugs are eliminated by first order kinetics and
are administered with identical dosing regimens. The volume of distributions of Drugs A and B
are identical. Which of the following statements is true?

a) steady state concentration of Drug A is equal to Drug B


b) drug A reaches steady state concentration earlier than Drug B
c) steady state concentration of Drug A is more than Drug B
d) drug A reaches steady state at the same time as Drug B
e) drug B reaches steady state concentration earlier than Drug A
C
Sixty percent (60%) of Drug A in the plasma is bound to albumin. Drug A distributes only into
the extracellular water. The extracellular water volume is 13 liters and the plasma volume is 3
liters. The plasma concentration is 10 mg/liter. Calculate the amount of drug A in the interstitial
volume.

a) 4mg
b) 12mg
c) 40mg
d) 56mg
e) 60mg
E
The following Figure represents the concentration of drug in plasma with time.

Which of the following statements best describes the drug characteristics during the times
shown?
a) the drug is eliminated by only first order elimination
b) the drug is eliminated by only zero order kinetics
c) the drug is eliminated by passive diffusion
d) the drug is eliminated by passive diffusion then facilitated transport
e) the drug is eliminated by facilitated transport
Error! Filename not specified.
C

Therapeutic index = LD50/ED50


Margin of safety = LD1/ED99
The figure above represents quantal dose-effect curves for a new drug. The curve A-B and C-D
represent therapeutic and toxic effects of this drug. Which of the following numbers
approximates the therapeutic index?

a) 0.5
b) 1.0
c) 1.5
d) 15
e) 50
Error! Filename not specified.
B
The figure above shows the elimination of drug x following administration of a 200 mg dose to a
50 kg woman. Calculate the elimination rate for the drug.

a) (0.1mg/liter)/hr
b) (1mg/liter)/hr
c) (10mg/liter)/hr
d) (100ug/liter)/hr
e) can not be calculated
Error! Filename not specified.
D
A drug is used to treat a patient suffering from nasal congestion. The drug produces its effect by
activating a G-protein coupled receptor that activates the G-protein Gq. The most likely
signaling pathway that this drug activates is:

a) adenylate cyclase and the production of cyclic AMP


b) adenylate cyclase and the production of inositol triphospahte
c) PI-Phospholipase C and the production of cyclic AMP
d) PI-Phospholipase C and the production of inositol triphosphate
e) adenylate phosphate and activation of protein kinase A
E, this has nothing to do with receptor affinity
Muscle strips are placed in a bath and exposed to drugs that elicit muscle contraction. The
curves above represent responses to several agonists that bind to the same receptor subtype.
The data indicate:

a) drug B has the greatest affinity for the receptor


b) drug B has the least affinity for the receptor
c) drug C is the most potent drug
d) drug A is less efficacious than C
e) drug B is the most potent drug
Error! Filename not specified.
C
A drug has been developed that passes the Blood brain barrier by Passive diffusion. A company
wishes to modify the drug so that more of the drug will cross the blood
brain barrier. The company should:

a) increase the binding of the drug to albumin


b) increase the charge of the molecule
c) increase the hydrophobicity of the drug
d) increase the size of the drug
e) increase the binding to albumin of the drug and its charge
E
Drugs A, B and C were administered i.v. to determine their pharmacologic properties. Drugs A
and B were administered at a dose of 10mg and C at a dose of 20mg.
Which of the following statements is true?

a) the volume of distribution for A is equal to C


b) the half-life of A is less than B
c) the half-life of C is greater than B
d) the volume of distribution for B is less than A
e) the volume of distribution for A is less than C
Error! Filename not specified.
E
The figure below shows the elimination of a drug following administration of a single
dose of 0.4mg/kg to a 50 kg woman. Calculate the volume of distribution (Vd).

a) 4 liters
b) 20 liters
c) 40 liters
d) 200 liters
e) 400 liters
Error! Filename not specified.
A
A drug is 50% absorbed and 80% of the drug is biotransformed in the liver. The transformed
form of the drug is not active. You administer 5mg of the drug orally 3 times a day. Calculate the
bioavailability of the drug.

a) 0.1
b) 0.2
c) 0.4
d) 0.5
e) 0.8
D
Drug A is to be administered I.V. to a healthy 60kg individual. The clearance of the drug is
20liters/hr, and the half-life is 10hrs. The therapeutic concentration is 0.2mg/liter. After steady
state has been achieved, what maintenance dose should be
administered?

a) 1mg every 2 hrs


b) 1mg every 4 hrs
c) 1mg every 12 hrs
d) 12mg every 3 hrs
e) 12mg every 12 hrs
B
A patient is on "Drug A" for pain. Drug A is a substrate of the CYP450 enzyme 2D6, and it is
metabolized to an inactive metabolite. Drug A is working well and the patient's
pain is under good control. The patient decides to take a second drug, "Drug B" for sinus
congestion. Drug B is an inducer of 2D6. What will be the potential clinical consequence of this
patient taking Drug B?

a) his pain will decrease.


b) his pain will increase.
c) the sinus medicine will not work.
d) he will have side effects related to the sinus medicine.
e) he will have side effects related to the pain medicine.
D
Mr. Jones is an 84 year old retired mailman who lives alone. He has several medical problems,
is on 12 medications, and admits to you (the physician) that "doc, sometimes I don't like to take
all those medicines, but I usually take most of them." Which factor is most important related to
this patient's risk of having an Adverse Drug Event?

a) age
b) social situation
c) his several medical problems
d) his number of medications
e) his compliance problem (the fact that he doesn't take the medications as prescribed)
A
Which pharmacokinetic parameter is least likely to change as a person ages?

a) absorption
b) distribution
c) biotransformation
d) excretion
D
A 39-year-old female is given a muscle relaxant prior to endotracheal intubation. Minutes later
the patient is noted to have a sudden onset of hyperthermia, tachycardia, sweating, cyanosis,
tachypnea and muscle rigidity. The muscle relaxant causing these
symptoms has the following mechanism of action:

a) competitive antagonist
b) cholinesterase inhibitor
c) antimuscarinic antagonist
d) depolarizing agent
e) muscarinic agonist
A
A pharmaceutical company has a promising drug that they think would be an advance in the
treatment of a disease. They have received permission to test their drug in normal volunteers.
This study would be classified as which type of trial?

a) phase I clinical study


b) phase II clinical study
c) phase III clinical study
d) phase IV clinical study
e) preclinical study
B
A 48-year-old man is diagnosed with Sjogren's Syndrome, which is characterized by chronic dry
mouth due to under secretion of saliva (Xerostomia). The drug you choose to treat this patient
acts on which type of receptor?

a) alpha2-adrenergic
b) muscarinic-cholinergic
c) nicotinic-cholinergic
d) beta1-adrenergic
e) beta2-adrenergic
C. Metaproterenol is a Beta-2 agonist used in the treatment of asthma and COPD.

Dobutamine is a beta-1 agonist, Clonidine is an alpha-2 agonist, Ephedrine is like longer acting
epi, dopamine would not be suitable
A 16-year-old athlete suffers an asthmatic attack after running for a touchdown. He is promptly
treated by using an inhaler and recovers in 10 minutes. Which of the following agents is most
likely to be the active ingredient in the inhaler?

a) clonidine
b) dobutamine
c) metaproterenol
d) ephedrine
e) dopamine
C
An elderly woman living in Kansas City suffers from primary open-angle (chronic simple)
glaucoma. She has been taking a medication once per day but upon visiting her sister in St.
Louis, she discovers she has forgotten her medication. You prescribe timolol and explain it most
likely works primarily by:

a) contracting the ciliary muscle


b) improves accommodation
c) inhibits the production of aqueous humor by the ciliary epithelium
d) relaxes the sphincter (circular) muscle of the iris
e) contracts the longitudinal ciliary muscle and pulls on the trabecular network to
relieve pressure
E
A woman suffering from myasthenia gravis is being treated with pyridostigmine. You observe
she is unable to continue climbing stairs after only a few minutes. A test dose of edrophonium
improved her ability to climb the stairs. This indicates that:

a) the dose of edrophonium was too large


b) the patient does not have myasthenia gravis
c) the dose of pyridostigmine should be reduced
d) the dose of edrophonium was insufficient
e) the dose of pyridostigmine should be increased
A. Relaxation of bronchial smooth muscle, beta-2.

Contraction of vascular smooth muscle is by alpha-1


Inhibition of insulin release from pancreatic islet cells is through alpha-2, while release is
through beta-2.
Contraction of radial muscle is through alpha-1.
Inhibition of NE release from sympathetic neurons is through alpha-2
Which of the following effects of epinephrine would be blocked by propranolol but not by
phentolamine?

a) relaxation of bronchial smooth muscle


b) contraction of vascular smooth muscle
c) inhibition of insulin release from pancreatic islet cells
d) contraction of the radial muscle of the iris
e) inhibition of NE release from sympathetic neurons
C
When a moderate dose of norepinephrine is given after pretreatment with a large dose of
atropine, which of the following is most likely?

a) a decrease in heart rate caused by a direct action


b) a decrease in heart rate caused by an indirect reflex action
c) an increase in heart rate caused by a direct action
d) an increase in heart rate caused by an indirect reflex action
e) no change in heart rate
A
Following a very exciting Pharmacology lecture you retire to the rest room. During the act of
voiding which of the following sequence of events takes place?

a) contraction of the detrusor muscle of the bladder, relaxation of the trigone muscle and
sphincter, relaxation of the external sphincter
b) contraction of both the detrusor and trigone muscle as well as the external sphincter
c) relaxation of the detrusor muscles and contraction of the trigone muscle and sphincter and
contraction of the external sphincter
d) contraction of the detrusor muscle only
e) relaxation of the detrusor and trigone muscle and relaxation of the external
sphincter
D. Oxybutynin, an m1,2,3 antagonist.

Ipratroprium is also a muscarinic ant, but it is poorly absorbed, and mostly used to treat COPD.
Atropine is a muscarinic antagonist that is pretty nonselective, and has lots of side effects. So
would scopolamine.
Physostigmine would produce the opposite effect.
A problem that approaches nearly 10% of elderly women is incontinence. This can be
embarrassing and affect one's life style. A drug that is commonly used to help with this problem
is which of the following:

a) ipratroprium
b) atropine
c) physostigmine
d) oxybutynin
e) scopolamine
E. Ipratroprium is a muscarinic antagonist used for first line treatment against COPD attacks.
Scopolamine is a muscarinic antagonist and so is atropine.
Tolterodine (Detrol) is a muscarinic antagonist usually used for urinary issues.
Physostigmine is a AChEI that would do the opposite.
Patients who smoke cigarettes often have a higher incident of chronic obstructive pulmonary
disease (COPD) which includes emphysema and bronchitis. Several types of drugs are
available to treat COPD. Which of the following would be preferred to treat acute bronchospasm
that can accompany COPD?

a) scopolamine
b) physostigmine
c) atropine
d) tolterodine
e) ipratropium
B. Trimethaphan, which is a nicotinic antagonist that would block fast EPSP.
Drug X has been given to a patient who developed a hypertensive crisis with a precipitous
severe elevation of blood pressure. Drug X produced the following effects: tachycardia,
cycloplegia, urinary retention, dry mouth, relaxation of arterial and venous smooth muscle and
pooling of blood. Drug X is most likely:

a) atropine
b) trimethaphan
c) datroline
d) pilocarpine
e) neostigmine
B
Drug X is most likely:
a) phentolamine
b) metoprolol
c) reserpine
d) clonidine
e) methyldopa
Error! Filename not specified.
E, an alpha-1 blocker
Drug Y is most likely:
a) reserpine
b) methyldopa
c) propranolol
d) clonidine
e) prazosin
Error! Filename not specified.
A
When norepinephrine is administered by a bolus iv injection of a therapeutic dose, which is the
most likely effect illustrated in the above table.
Error! Filename not specified.
A???
A drug which would cause the appearance of the direct effect of norepinephrine on heart rate
could be:

a) phentolamine
b) propranolol
c) isoproterenol
d) atropine
e) albuterol
C
Botulinium toxin (BoTox) has been found useful in treating several ocular conditions such as
strabismus (cross-eyes), hemifacial spasms), muscle spasms such as cervical dystonia;
achalasia) as well as dermatological conditions such as hyperhidrosis. This drug works by which
of the following mechanisms?

a) blockade of the release of intracellular Ca2+ from the sacroplasmic reticulum


b) blocking the nicotinic-muscarinic receptor
c) preventing the neuronal release of acetylcholine
d) blocking acetylcholinesterase
e) blockade of the uptake of acetylcholine into synaptic vesicles
C. Atropine blocks muscarinic receptors, while pralidoxime regenerates irreversibly inhibited
AChE
A patient is brought into the Emergency Department after being exposed to a chemical she was
using on plants, weeds and flowers in her backyard. She presented with the following
symptoms: ciliary spasm, miosis, bronchoconstriction, sweating, salivation, bradycardia, muscle
weakness, confusion and ataxia. You quickly decide to treat pharmacologically. The wisest
choice would be which of the following?

a) atropine
b) pralidoxime
c) atropine + pralidoxime
d) epinephrine + atropine
e) ipratroprium + atropine
B
In comparing methyldopa and guanethidine which of the following is correct?

a) guanethidine - but not methyldopa - results in salt and water retention when used alone
b) guanethidine causes fewer CNS side effects (such as sedation) than methyldopa
c) guanethidine causes more immunologic adverse effects (e.g. hemolytic anemia)
than methyldopa
d) guanethidine is less efficacious than methyldopa in severe hypertension
e) methyldopa causes more orthostatic hypotension; than guanethidine
A. Phenylephrine is an alpha-1 agonist, leads to increases in SVR and reflex bradycardia.
Phentolamine is an alpha blocker that would reduce this effect.
When given to a patient, phentolamine blocks which of the following?

a) bradycardia induced by phenlyephrine


b) bronchodilation induced by epinephrine
c) increased cardiac contractile force induced by norepinephrine
d) miosis induced by acetylcholine
e) vasodilation induced by isoproterenol
A. Bethanachol can cause muscarinic stimulation on nerve terminals in vasculature, leading to
decrease in SVR and reflex tachycardia. Propranolol will block this effect by mitigating
sympathetic reflex responses.
You are carrying out a series of experiments examining the effect of autonomic drugs and the
responses they produce in an anesthetized mammal. You determine that propranolol will block
which of the following?

a) bethanachol induced tachycardia


b) nicotine-induced hypertension
c) norepinephrine-induced bradycardia
d) phenlyephrine-induced mydriasis
e) pilocarpine-induced miosis
C. Terazosin is an alpha-1 blocker
The above diagram depicts the effect of various drugs on blood pressure in an anesthetized
mammal before and after pretreatment with another drug (Drug Z).

Drug Z is most likely:


a) trimethaphan
b) cocaine
c) terazosin
d) metoprolol
e) reserpine
Error! Filename not specified.
D. Phenylephrine
The above diagram depicts arterial pressure, and nerve impulses in the carotid sinus nerve, the
vagus nerve, sympathetic cardiac nerve, and sympathetic vasoconstrictor nerve from an
unanesthetized human.
23. Which of the following drugs given i.v. would result in situation A?
a) prazosin
b) acetylcholine
c) histamine
d) phenlyephrine
e) neostigmine
Error! Filename not specified.
A
The above diagram depicts arterial pressure, and nerve impulses in the carotid sinus nerve, the
vagus nerve, sympathetic cardiac nerve, and sympathetic vasoconstrictor nerve from an
unanesthetized human.

Which of the following drugs given i.v. would result in Situation B?


a) acetylcholine
b) atropine
c) norepinephrinie
d) prazosin
e) epinephrine
Error! Filename not specified.
C
The enzyme (ENZYME W) involved in the synthesis of neurotransmitter 2 is:

a) acetylcholinesterase
b) tyrosine hydroxylase
c) choline acetyltransferase
d) choline hydroxylase
e) monoamine oxidase
Error! Filename not specified.
C. Ambenonium is a cholinesterase inhibitor
A drug that acts on Enzyme Z to break down neurotransmitter 2 could be:
a) atropine
b) bethanechol
c) ambenonium
d) succinylcholine
e) vecuronium
Error! Filename not specified.
D
In comparing metoprolol and propranolol which of the following is a correct statement?

a) propranolol but not metoprolol displays intrinsic sympathomimetic action (partial


agonism)
b) propranolol but not metoprolol has been found effective in treating congestive
heart failure
c) metoprolol but not propranolol also blocks α-adrenergic receptors in addition to β- adrenergic
receptors
d) metoprolol but not propranolol displays β1-selexctive blocking properties
e) both metoprolol and propranolol are contra-indicated in treating asthmatic
patients
C
Which of the following drugs is correctly matched with a major side effect likely to be produced?
(Drug: Side Effect)

a) atropine : cyclospasm and miosis


b) neostigmine : mydriasis
c) phenoxybenzamine : postural (orthostatic) hypotension
d) pindolol : tachycardia
e) bethanechol : urinary retention
D
Which of the following drugs is correctly matched with its presumed mechanism of action (Drug :
Mechanism of Action)

a) edrophonium : muscarinic-cholinergic antagonist


b) scopolamine : cholinesterase inhibitor
c) pilocarpine : inhibition of acetylcholine synthesis
d) ephedrine : mixed acting sympathomimetic
e) clonidine : directly acting α1-adrenergic agonist
A
A patient has been treated with reserpine for three weeks for hypertension. Her physician
(who was not trained at Saint Louis University School of Medicine) decides to treat the patient
with a decongestant (e.g. phenlyephrine). The patient experiences severe
hypertension. The reason for such a response is:

a) upregulation of postjunctional α1-adrenergic receptors


b) blockade of norepinephrine reuptake across neuronal membranes
c) marked increase in the release of norepinephrine from adrenergic varicosities
d) inhibition by reserpine of the biotransformation of phenlyephrine
e) enhancement of phenylephrine's effect by tyramine due to inhibition of
monoamine oxidase by reserpine
A
In the figure above, changes in bronchomotor tone are depicted in terms of constriction (+) or
relaxation (-). the first panel of the figure (control) shows changes that were elicited by five
different procedures (numbered 1 through 5), each of which was performed alone. The
remaining panels show changes that were elicited by the same five procedures, each of which
was performed after pretreatment with a different drug. Based on the results, procedure 1 is
most likely the administration of:

a) administration of bethanechol
b) administration of histamine
c) administration of isoproterenol
d) stimulation of sympathetic nerves
e) stimulation of the vagus nerve
Error! Filename not specified.
D. Carvedilol is, along with labetalol, a non selective beta blocker with also alpha-1 blocker
activities.
A new drug was administered to an anesthetized animal with the results shown. A large
dose of epinephrine was administered before and after the new agent for comparison.
Which of the following agents does the new drug most closely resemble?

a) nadolol
b) scopolamine
c) prazosin
d) carvedilol
e) metoprolol
Error! Filename not specified.
C
Of the following pairs of drugs and the CYP450 enzyme that carries out
biotransformation which pair is not correctly matched?

a) polycyclic aromatic hydrocarbon CYP1A1


b) ethanol CYP 2E1
c) cimetidine CYP4A2
d) propranolol CYP 2D6
e) codeine CYP 2D6
E.

Phase I are mostly nitro reductions, oxidations, deaminations, dealkylations, and hydroxylations.
Phase II are conjugation rxns: glucuronidations, acetylations, mercapturic acid formation, sulfate
conjugation, N O and S methylation, trans-sulfuration
Biotransformation reactions have been conveniently placed in two types; namely phase 1 and
phase 2. Which of the following is a phase 2 reaction?

a) nitro reduction
b) deamination of epinephrine
c) o-dealkylation
d) aliphatic hydroxylation
e) acetylation
A. Phenoxybenzamine is an alpha blocker
The following diagram depicts the effects of various drugs on blood pressure on an anesthetized
mammal before and after pretreatment with Drug X.

Drug X is:
a) phenoxybenzomine
b) propranolol
c) atropine
d) neostigmine
e) reserpine
Error! Filename not specified.
B. Atropine
The following diagram depicts the effects of various drugs on blood pressure on an anesthetized
mammal before and after pretreatment with Drug Y.

Drug Y could be:


a) propranolol
b) atropine
c) reserpine
d) clonidine
e) α-methyldopa
Error! Filename not specified.
C. Grapefruit juice and cimetidine are inhibitors
Grapefruit juice and cimetidine are known to contain substances that affect CYP 3A4 enzymes.
If Drug A is metabolized by CYP 3A4, which of the following is most likely to happen to Drug A:

a) there will be an increase in the rate of biotransformation


b) there will be an increase in the rate of production of metabolites
c) there will be a decrease in pharmacological effects if the metabolites are active
d) there will be a decrease in the serum half life
e) there will be a decrease in serum and total free drug concentration
A
Which pharmacokinetic parameter is least likely to change as a person ages?

a) absorption
b) distribution
c) hepatic metabolism
d) renal excretion
A. Isoniazid is transformated mainly by n-acetyation.
Isoniazid is an important anti-tubercular drug that can result in drug interactions. It is
biotransformed primarily by which of the following
a) N-acetylation
b) ester hydrolysis
c) oxidation
d) glycine conjugation
e) dehydrogeneration
D
A patient is admitted to the emergency room 4 hours after taking an overdose of barbiturate.
The plasma concentration of the drug at the time of admission is 100mg/liter, and the volume of
distribution and the half-life of the drug are 30liters and 2 hours, respectively. The ingested dose
was?

a) 25mg/liter
b) 50mg/liter
c) 200mg/liter
d) 400mg/liter
e) 800mg/liter
C
A patient suffering from depression and is allergic to SSRI drugs (selective serotonin reuptake
inhibitors) and tricyclic antidepressants was administered a monoamine oxidase inhibitor
(MAOI). You inform this patient that certain foods containing tyramine could now be absorbed
and could cause dangerously elevated blood pressure. The primary mechanism of action of
tyramine is a:

a) direct action on α1-adrenergic receptors


b) direct action on β1-adrenergic receptors
c) release of norepinephrine from sympathetic varicosities
d) stimulation of gut and liver catechol-o-methyl transferase
e) blockade of muscarinic-cholinergic receptors
E
The pharmacokinetics of drugs A, B and C are shown in table below. The data was obtained
with a healthy male volunteer (70kg) following both i.v. and oral administrations of 10mg doses
for each drug. Which of the following statements is true?

a) bioavailability of drug A is equal to drug B


b) bioavailability of drug A is greater than drug B
c) bioavailability of drug B is equal to 1
d) bioavailability of drug A is equal to drug C
e) bioavailability of drug C is equal to 0.5
Error! Filename not specified.
D. Inhibitor of the Gi receptor, that would mean increase in cAMP and thus increase in PKA
activity
You are studying a G protein coupled receptor and find that it activates Gi. You have
generated an inhibitor to the receptor. Which of the following down stream effects do
you expect to occur when your inhibitor is acting on the receptor?
a) increase of cyclic GMP
b) decrease of cyclic AMP
c) decrease of cyclic GMP
d) increase of PKA activity
e) increase of 1,4,5 inositol triphosphate
C
A 43-year-old female with thyroid dysfunction is currently thought to be in thyroid storm
(thyrotoxicosis). She is given a drug to prevent cardiovascular effects seen in thyrotoxicosis.
The drug she was given most likely blocks which of the following receptors?

a) alpha1-adrenergic
b) muscarinic cholinergic
c) beta adrenergic
d) nicotinic cholinergic
e) alpha2-adrenergic
C

Dm = (Css Cl Tm)/F
Dm = (1 L/Hr 10 mg/LTm)/(0.40)
Dm/Tm = 25 mg/Hr
A patient (55kg) is taking a drug orally to reduce blood pressure. The clearance of the drug is
1.0liters/hour. The volume of distribution is 10ug/ml and the drug is 40% absorbed and does not
undergo biotransformation. What dosing regimen should be
administered to attain a therapeutic concentration of 10mg/liter?

a) 150mg every 10 hour


b) 250mg every 6 hours
c) 250mg every 10 hours
d) 150mg every 4 hours
e) 25mg every 2 hours
C
A drug was present in the plasma shortly after i.v. administration at a concentration of 1mg/ml.
Ten hours later, the plasma concentration was determined to be 250ug/ml. The drug is
eliminated by the kidney by first order kinetics. The half-life of the drug is:

a) 2 hours
b) 3 hours
c) 5 hours
d) 8 hours
e) 10 hours
B
A patient is on "Drug A" for pain. Drug A is a substrate of the CYP450 enzyme 2D6, and it is
metabolized to an inactive metabolite. Drug A is working well and the patient's pain is under
good control. The patient decides to take a second drug, "Drug B" for sinus congestion. Drug B
is an inducer of 2D6. What will be the potential clinical consequence of this patient taking Drug
B?

a) his pain will decrease


b) his pain will increase
c) the sinus medicine will not work
d) he will have side effects related to the sinus medicine
e) he will have side effects related to the pain medicine
C
A company has determined the efficacy, potency and toxicity of a drug. The dose dependency
curves are shown above. Calculate the therapeutic index of the drug.

a) 10
b) 50
c) 100
d) 200
e) 400
Error! Filename not specified.
D. Involves trapping the drug in ionized form in urine.
You are treating a patient with a drug that is a weak base. The pK of the drug is 8.6 and its
molecular weight is 205. The drug is eliminated exclusively in the kidney by passive diffusion.
The extent of urinary excretion of the weak base will increase when:

a) The urine is more basic than the plasma


b) The pH of the urine is equal to the pK of the drug
c) The pH of the plasma is equal to the pK of the drug
d) The urine is more acidic than the plasma
e) The urine and plasma have the same pH
B
You, the physician, are taking care of an 84 year old frail woman in the hospital. If your concern
is her risk of adverse drug events (ADEs), which is the most important factor
associated with her risk of ADEs?

a) her age
b) her number of medications
c) the type of medications she is prescribed
d) the pharmacodynamics of the drugs she is prescribed
e) the pharmacokinetics of the drugs she is prescribed
D
A new drug (drug X) is an antagonist against a beta1 adrenergic receptor. Additions of
increasing concentrations of norepinephrine overcome the effect of the drug so that values of
Emax are reached that equal those found in the absence of the drug. Drug X can be classified
as:

a) a chemical antagonist
b) a physiological antagonist
c) an irreversible non-competitive antagonist
d) a reversible competitive antagonist
e) an irreversible competitive antagonist
D
Disulfiram has been used to treat patient suffering from severe alcoholism. In the presence of
disulfiram, ethanol consumption results in patients being very sick. They experience nausea,
vomiting, dyspnea and occasional hypotension. The nausea, vomiting
and dyspnea is due to inhibition of:

a) CYP 2D6
b) alcohol dehydrogenase
c) monoamine oxidase
d) aldehyde dehydroxygenase
e) glutathione dehydrogenase
B
The pharmacological effect of drugs acting on different membrane receptors occur at quite
different time scales. Which of the following membrane receptors has the fastest time scale in
mediating its pharmacological effect?
a) nuclear receptors
b) ion channels
c) G-protein coupled receptors
d) tyrosine kinase receptors
e) serine/thereonine receptors
D. 280 mg

D(L) = (Vd * Css) / F


A patient (70 kg male) is admitted to the hospital with a serious bacterial infection resulting in
pneumonia. You need to administer a loading dose to achieve rapidly the therapeutic plasma
concentration of 4mg/liter. Vd and clearance of the drug are 1000 ml/kg and 50ml/min,
respectively. What loading dose should you administer?

a) 4mg
b) 40mg
c) 28mg
d) 280mg
e) 320mg
B
You have administered 20mg of a drug i.v. and collected the data shown below. Calculate the
volume of distribution using the graph below.

a) 4 ml
b) 3.3liters
c) 4liters
d) 13.3liters
e) 400ml
Error! Filename not specified.
D

Phenylephrine is an alpha-1 agonist. It would trigger a change in heart rate through baroreflex
mechanisms.
Drug X causes an increase in blood pressure and a decrease in heart rate when administered
intravenously. If an antagonist at ganglionic nicotinic receptors is administered first, drug X
causes an increase in blood pressure and no change in heart rate. Drug X is most likely:

a) epinephrine
b) dopamine
c) norepinephrine
d) phenlyephrine
e) dobutamine
D
A drug company has just concluded drug tests with two drugs (drug 1 and 2) that lower blood
pressure. The figure above represents the quantal dose-effect curves.

a) drug 1 is more efficacious and more potent than drug 2


b) drug 1 and 2 are equally efficacious but drug 2 is more potent
c) drug 1 and 2 are equally efficacious but drug 1 is more potent
d) drug 1 is more efficacious and equally potent to drug 2
e) drug 2 is less efficacious but the same potency as drug 1
Error! Filename not specified.
D
Your patient has been admitted to the hospital with pneumonia. You decide to administer i.v. an
antibiotic. The clearance and half-life of the antibiotic are 3liters/hour and 5 hours, respectively.
After administering a bolus dose to obtain a therapeutic concentration of 8mg/liter, you
administer a maintenance dose. What maintenance dose must be
administered every 5 hours.

a) 3mg
b) 12mg
c) 30mg
d) 120mg
e) 300mg
C. Tacrine is a AChEI used to treat Alzheimer's disease
Acetylcholinesterase inhibitors have been used to improve the symptoms of Alzeheimer's
Disease, a devastating disease of the elderly. This includes which of the following?

a) neostigmine
b) pralidoxime
c) tacrine
d) pyridostigmine
e) parathion
B
The company you work for has developed a new drug. You have determined that 90% of the
drug binds to serum proteins. The plasma concentration of the drug is 100mg/liter. The
extracellular and plasma volumes are 11 and 3 liters, respectively.
The amount of drug in the interstitial volume is:
a) 30mg
b) 80mg
c) 110mg
d) 200mg
e) 300mg
D. Resting state has GDP bound. Binding makes GDP release, allowing binding of GTP, then G
proteins dissociate. GTP hydrolysis to GDP makes G protein couple with the receptor again.
A new G-protein coupled receptor that regulates heart rate has just been isolated. Your
technician characterizes the receptor. Which of the characteristic concerning your receptor is
true?

a) the receptor is not saturable


b) binding of an agonist to the receptor results in the binding of GDP to G-proteins
c) binding of an agonist to the receptor activates the Jaks-Stat signaling pathway
d) binding of an agonist to the receptor to the receptor results in release of GDP from
G-proteins
e) binding of an antagonists to the receptor results in the binding of GTP to G proteins
A
Drug A is a weak organic base with pK of 8.5. What percent of the drug is in the unionized form
in the small intestine (pH = 5.5)?

a) less than 1%
b) 1%
c) more than 1%, less than 10%
d) 10%
e) 90%
E
A 50kg patient is given a hypnotic i.v. If the hypnotic drug is infused continuously at the rate of
1mg/kg/hr, and the steady state concentration is attained in 10 hours the half-life of the drug is:

a) 20 minutes
b) 30 minutes
c) 1 hour
d) 1 hour 30 minutes
e) 2 hours
C, Carvedilol along with Labetalol (two 3rd gen beta blockers) also have alpha-1 antagonist
activity.

FYI, 3rd gen b1-blocker with beta-2 AGONIST activity is Celiprolol


A 24 year old woman in her 25th week of pregnancy is admitted to the hospital because of
preeclampsia, with manifestations of visual disturbances, headache, increasing blood pressure,
proteinuria, and rapid development of edema. To control the patient's blood pressure, you as
her physician decide to use a drug that is a potent blocker of both alpha- and beta-adrenergic
receptors. Which of the following drugs would you most likely use:

a) timolol
b) carvedilol
c) propranolol
d) pinodol
e) prazosin
C
Two drugs A and B that are competitive inhibitors of acetylcholine are being compared. Drug A
is 75% absorbed in the gut and is not metabolized in the liver. Drug B is 90% absorbed in the
gut and 80% metabolized in the liver. Only the unmetabolized drug B is
active. Which of the following statements concerning drugs A and B is true?

a) the bioavailability of drug A is equal to drug B


b) the bioavailability of drug A is less than drug B
c) the bioavailability of drug A is greater than drug B
d) do not have sufficient data to determine which drug has the higher bioavailability
C
The hematocrit and the plasma concentration of drug A are 0.4 and 10mg/liter, respectively.
Calculate the blood concentration, if drug A is uniformly distributed.

a) 2mg/liter
b) 4.2mg/liter
c) 8.8mg/liter
d) 10mg/liter
e) 12.8mg/liter
D
The graph above is the plasma concentration versus time plot of drug A. Using the graph which
of the following statements is true?

a) the half-life is 5 hours


b) the elimination rate is 20ug/hour
c) the half-life is 4 hours
d) the elimination rate is 100ug/hour
e) the elimination rate is 200ug/hour
Error! Filename not specified.
A
Your patient is taking a drug orally. The patient says that it is difficult to take the drug as many
times a day that you have prescribed. Which change in drug administration will allow you to
decrease the number of times a day the drug is given without changing the steady state?

a) increase the dose


b) decrease the dose
c) change the drug formulation to decrease bioavailability
d) change the formulation to decrease absorption
e) none of the above
E
Benign prostatic hyperplasia is a condition that affects numerous males as they age with an
incidence of 60% at age 70 and nearly 90% at age 80. Tamulosin is a pharmacological agent
used to treat this condition. Tamulosin exerts its beneficial effect by acting on which of the
following receptors:

a) beta 1-adrenergic
b) muscarinic-cholinergic
c) alpha 2-adrenergic
d) beta 2-adrenergic
e) alpha 1-adrenergic
D. The higher the partition coefficient, the better it can pass over membranes, the more the drug
is absorbed.
Drug X is a weak acid and can be taken orally. The drug is absorbed in the small intestine by
passive diffusion. To increase the rate of uptake the company should modify the drug to:

a) decrease its partition coefficient


b) increase the amount ionized in the intestine
c) decrease its pK
d) increase its partition coefficient
e) none of the above
B. Meperidine should not be taken in the elderly.
A robust appearing 88 year old man (still plays tennis and golf), is admitted to the hospital
because he fell of his bike and sustained a fracture of his right femur. He is in excruciating pain.
While waiting for his surgery, you start him on a narcotic drug. Of all the narcotics available for
this amount of pain (moderate to severe), which one should be
avoided?

a) fentanyl
b) meperidine
c) morphine
d) oxycodone
D. Activation of alpha-1 receptors leads to activation through Gq/11 and PLC --> Dag and IP3
--> increasein Ca2+
A 69 year old man receives an infusion of norepinephrine to increase blood pressure following a
traumatic spinal cord injury. The signal transduction mechanisms leading to this effect include:
a) activation of a tyrosine kinase receptor
b) inhibition of adenylyl cyclase
c) activation of guanylyl cyclase
d) activation of phospholipase C
e) activation of protein kinase A
E. phosphorylation of receptors
A 72 year old woman is treated for acute heart failure with an infusion of
dobutamine. The duration of dobutamine infusion should be limited to several hours per day
because prolonged infusion may lead to drug tolerance as a result of:

a) induction of drug metabolizing enzymes


b) depletion of signal pathway components
c) degradation of G protein
d) phosphorylation of receptors
e) upregulation of receptors
D
Propoxyphene is the narcotic compound in medicines such as Darvon and
Darvocet. Why is it on a list of medicine considered "inappropriate" for the
elderly?
a) The risk of falls is higher than other narcotics
b) The incidence of constipation is higher than other narcotics
c) The incidence of drowsiness is higher than other narcotics
d) The analgesic effect is not much better than acetominophen
e) The analgesic effect is not much better than other narcotics
E. Increase acetylcholinesterase activity.

Pralidoxime knocks organophosphates off of acetylcholinesterase, regenerating the enzyme.


A 34 year old man presents to the emergency department complaining of salivation, diarrhea
and labored breathing shortly after dermal and inhalational exposure to an organophosphate
product in his delivery truck. Pralidoxine is
included in his treatment because of its ability to:

a) block nicotinic acetylcholine receptors


b) block muscarinic acetylcholine receptors
c) inhibit synthesis of acetylcholine
d) prevent organophosphate binding to the anionic site of cholinesterase
e) increase acetylcholinesterase activity
C
a 38 year old woman undergoes a routine ophthalmologic evaluation that includes examination
of the peripheral retina. Topical ocular administration of either phenylephrine or homatropine (an
analogue of atropine) both may cause

a) cycloplegia
b) miosis
c) mydriasis
d) miosis and cycloplegia
e) mydriasis and cycloplegia
C
A 29 year old woman with myasthenia gravis is being treated with a steroid (prednisone) and
pyridostigmine. She complains to her physician of increasing diplopia and facial weakness. If an
intravenous dose of edrophonium causes increased muscle weakness in this patient, it indicates
that:

a) the diagnosis is incorrect


b) the dose of pyridostigmine should be increased
c) the dose of pyridostigmine should be decreased
d) atropine should be added to the regimen
e) prednisone should be discontinued
B
A 59 year old man with chronic open angle glaucoma is placed on topical ocular drug therapy
for the condition. In this patient both timolol and a praclonidine (an
analogue of clonidine) would cause:
a) increased cyclic AMP levels in the ciliary process
b) decreased cyclic AMP levels in the ciliary process
c) activation of adrenergic receptors linked to Gi proteins
d) blockade of adrenergic receptors linked to Gs proteins
e) increased outflow of aqueous humor of Schlemm's canal
C
An 83 year old patient is brought to your office by her daughter. The patient is on 14
medications, and she believes some of the medicines are causing a lot of her problems, and
wants to stop as many as possible. Her daughter however, wants "the most benefit" that new
medicines have to offer. What three guiding principles or measures should you use to evaluate
the "benefits" of medicines for this patient?
a) Number of medications, number of illnesses, life expectancy
b) Number of illnesses, life expectancy, comprehensive geriatric assessment
c) Life expectancy, comprehensive geriatric assessment, prioritize outcomes
d) Comprehensive geriatric assessment, prioritize outcomes, number of
medications
e) Prioritize outcomes, number of medications, number of illnesses
D, such as Tamsulosin
A 63 year old man complains of urinary urgency and frequency and his condition is found to be
caused by benign prostatic hyperplasia. These symptoms could be
effectively relieved by administration of an agent that:
a) activates muscarinic receptors
b) blocks muscarinic receptors
c) activates adrenergic α1 receptors
d) blocks adrenergic α1 receptors
e) activates cholinergic nicotinic receptors
E. Phenoxybenzamine, used in tx of pheochromocytoma
A 33 year old man experiences episodic attacks of severe hypertension caused by an adrenal
medullary tumor that secretes copious amounts of epinephrine and norepinephrine, and he is
scheduled for surgery to remove the tumor. Which of the following agents would shift the dose
response curve for norepinephrine to
the right and decrease the maximal effect in this patient?
a) propranolol
b) clonidine
c) prazosin
d) phentolamine
e) phenoxybenzamine
E
When given to a patient that has been diagnosed with angina pectoris propranolol would block
which of the following?
a) bradycardia induced by phenylephrine
b) miosis induced by acetylcholine
c) bronchoconstriction induced by isoproterenol
d) bronchoconstriction induced by pilocarpine
e) tachycardia induced by acetylcholine
B
You have found that drug A acts through a receptor that results in an Emax with drug A bound
to only 10% of receptors. Which of the following statements concerning this receptor is correct?

a) the EC50 and Kd of drug A are identical


b) the EC50 is less than Kd for drug A
c) the EC50 is greater than Kd for drug A
d) in the absence of any drug the receptor activates its downstream effectors
e) the EC50 for drug A can not be determined
B, through blocking baroreflex mechanism
You are carrying out a series of experiments examining the effect of autonomic drugs and the
responses they produce in a lightly anesthetized mammal. You determine that atropine will
block which of the following?
a) bethanechol induced tachycardia
b) phenylephrine induced bradycardia
c) norepinephrine induced mydriasis
d) nicotine induced hypertension
e) ephedrine induced CNS stimulation
C
A patient comes into your office and upon examination you observed the
following side effects: mydriasis, constipation, hot flushed skin, dry mouth, blurred vision, and
urinary retention. The patient had been taking medication but
couldn't remember the name. Your best estimate was the patient was taking
which of the following class of drugs:
a) a beta blocker
b) α1 adrenergic agonist
c) a muscarinic acetylcholine antagonist
d) an α-adrenergic antagonist
e) a β2-adrenergic agonist
B
The pK of a very weak acid is 5.4. What % of the drug will be in the lipid insoluble
form in the plasma pH = 7.4?
a) 1%
b) 99%
c) 90%
d) 10%
e) 9%
E. Epherine is like longer acting epinephrine
While on a camping trip in Southern Utah, you find a plant that your partner informs you is used
to make "Mormon Tea." Since you carried your Goodman and Gilman with you so you could
review your pharmacology, you note that this plant is similar to one used in Ancient China called
"Ma Huang." You correctly tell your partner that to block the pharmacological effects of the
active ingredient of this plant you would need the following:

a) a beta adrenergic antagonist


b) an alpha adrenergic antagonist
c) a muscarinic acetylcholine antagonist
d) a muscarinic acetylcholine antagonist plus an alpha adrenergic antagonist
e) an alpha adrenergic antagonist plus a beta adrenergic antagonist
17. C
18. C
19. C
20. B
The above diagram depicts targets for drug action (A - E) located in nerves or innervated
organs. Match the following drugs with their primary site of action (Letter can
be used more than once).
17. carvedilol
18. metaproterenol
19. doxazosine
20. vecuronium
Error! Filename not specified.
E. Alpha-1 receptors in the iris control mydriasis, so blocking adrenergic receptors would
possibly account for the left sided treatment with tyramine (a catecholamine release agent).
However, since mydriasis is still instigated by epi (and in fact it is a little bit bigger), adrenergic
blockade is probably not happening.
Tyramine is a catecholamine release agent, so it is dependent on catecholamines that are
already in the nerve. Reserpine (VMAT inhibitor) depletes vesicular levels of catecholamines.
This accounts for no response with tyramine, but response seen with epi.
The circles represent the size of a patient's pupils without treatment and following
treatment with amphetamine and with epinephrine. Which of the following is
compatible with the findings shown in the left eye?
a) blockade of β-adrenergic receptors
b) blockade of α-adrenergic receptors
c) blockade of muscarinic acetylcholine receptors
d) inhibition of cholinesterase
e) three day pre-treatment of the patient with reserpine
Error! Filename not specified.
E
Following the administration of a large dose of vasodilator sufficient to produce a marked
decrease in mean arterial blood pressure. Changes in the firing rate of
each of the following nerves is most likely:
a) the firing rate of the postganglionic sympathetic vasoconstrictor nerve
would increase
b) the firing rate of the carotid sinus nerve would increase
c) the firing rate of the postganglionic sympathetic cardiac nerve would
decrease
d) the firing rate of both the preganglionic and postganglionic vagus nerve
would increase
e) the firing rate of the preganglionic sympathetic cardiac nerve would
decrease
B. 10%, because only 40% is absorbed, and 75% of that absorbed drug is biotransformed,
leaving 10% that is bioavailable (i.e. absorbed into the plasma).
Drug A is taken orally and is 40% absorbed and 75% biotransformed. The
therapeutic dose of the drug is 10mg/liter. The half-live of the drug is 2 days and
clearance is 0.1liters/4 hours. Interstitial volume is 8 liters and the plasma volume
is 3 liters. The % of the drug A that then reaches the plasma is?
a) 5%
b) 10%
c) 20%
d) 50%
e) 80%
D. Beta bllockers cause vasoconstriction by blocking beta-2 receptors in the lung.
Beta adrenergic antagonist have been found useful in treating a wide variety of
conditions. These are powerful drugs and side effects do occur. Which of the
following is of the greatest concern?
a) decrease in heart rate in a patient with angina pectoris
b) membrane stabilizing effects in a patient with pheochromocytoma
c) fatigue and lethargy in patients with migraine headaches
d) bronchoconstriction in COPD patients
e) decreases in intraocular pressure in glaucoma patients
E. Phenoxybenzamine is an alpha-blocker (with some selectivity for alpha-2) that is associated
with all of those side effects, especially orthostatic hypotension. But Guanethidine does all of the
same, and does not enter the CNS.
An elderly physician that you know well is treating a patient with essential hypertension. Try as
you might you cannot convince him to use some of the more modern drugs in his patients. He
has prescribed a drug that produces the
following side effects: orthostatic hypotension, increased gastrointestinal motility, diarrhea,
miosis, bradycardia but no CNS effects. The drug is most likely:
a) propranolol
b) reserpine
c) nadolol
d) phenoxybenzamine
e) guanethidine
E. Succinylcholine is not metabolished by AChE, but rather by butrylcholinesterase (plasma
cholinesterase).
Following abdominal surgery which included the neuromuscular blocking agent,
succinylcholine, to produce adequate skeletal muscle relaxation, patient X
experienced prolonged apnea. Patient X most likely had a genetic defect in which
of the following enzymes?

a) glucose-6-phosphate dehydrogenase
b) cytochrome P450
c) UDP-glucuronyl transferase
d) N-acetyl transferase
e) plasma cholinesterase
C. Disulfiram inhibits aldehyde deH and thus leads to build up of acetaldehyde.
Chronic alcoholics are sometimes treated with a drug that results in a series of
violent unpleasant effects including flushing, dyspnea, nausea, vomiting and
hypotension. The mechanism of action of this drug is to inhibit:

a) CYP3A4
b) monoamine oxidase
c) aldehyde dehydrogenase
d) CYP1A2
e) alcohol dehydrogenase
D.
A patient is taking drug A orally that gives a plasma drug concentration of
10ug/ml. Drug A is 90% bound to serum albumin. The patient now is put on a
new drug such that: The total amount of drug A in plasma is unchanged but a
fraction of drug A is displaced from albumin. In the presence of the two drugs
only 80% of drug A is bound to albumin. After administration of the second drug:

a) the concentration of total drug A in the plasma will be 20ug/ml


b) the amount of free drug in the plasma does not change
c) the half life of drug A increases
d) the concentration of free drug A doubles
e) the concentration of free drug in the plasma decreases
C
A condition of decreased responsiveness to a drug which is acquired after prior or repeated
exposure to a given drug or one that is closely related in pharmacological activity is called
tolerance. Which of the following is not an example of pharmacodynamic or cellular tolerance:
a) homologous desensitization
b) internalization of a receptor
c) enzyme induction
d) heterologous desensitization
e) receptor phosphorylation
D, also B?
Bethanechol is administered subcutaneously to a patient with postoperative abdominal
distention and gastric atony. The subcutaneous route of
administration is chosen over the oral route because gastric retention is complete and there is
no passage of gastric contents into the duodenum. Which of the following effects is likely to be
observed after the subcutaneous administration of
bethanechol?
a) skeletal muscle paralysis
b) increase in heart rate
c) peripheral vasoconstriction
d) bronchoconstriction of the lung
e) dry mouth
B
An elderly woman is found to exhibit elevated intraocular pressure, and openangle glaucoma is
diagnosed. Her physician prescribes instillation into the eye of pilocarpine ophthalmic solution
every 6 hours. The anticipated effect of pilocarpine eye drops would be to:
a) relax the ciliary muscles
b) constrict the pupil
c) relax the sphincter muscle of the iris
d) inhibit the production of aqueous humor
e) increase the intraocular pressure
C
A patient you have been treating for hypertension by prescribing propranolol, suddenly stops
taking his medication and develops heart palpitations and rhythm
irregularities. What receptor mechanism might explain his current problem.

a) chronic β-blockers caused downregulation of cardiac β-receptors


b) β-blockers are still in his blood stream exerting an effect one day after
stopping
c) chronic β-blocker treatment caused supersensitivity of β receptors
d) he should have been on another type of antihypertensive medication
e) over time, β-blocker treatment caused desensitivity of β receptors
B
An advertisement in a local newspaper seeks to enroll 200 patients with arthritis
in a medical study that would be the first time that a new drug would be tested in
persons with this disease. The study would therefore be classified as which type
of trial?

a) phase I clinical study


b) phase II clinical study
c) phase III clinical study
d) phase IV clinical study
e) phase 0 clinical study
D. Physostigmine was the first AChEI to be used to treat myanthenia gravis, but it has been
replaced by more effective quarternary ammonium compounds: neostigmine, ambenonium, and
pyridostigmine
A 25 year old woman presents with muscle weakness, drooping eyelids and double vision that
she states has worsened over the past 3-4 months. These
symptoms are rapidly reversed when she is given iv edrophonium for diagnostic
purposes. Which of the following agents might best treat her disease?
a) bethanechol
b) physostigmine
c) pilocarpine
d) pyridostigmine
e) atropine
A
A 55 year old man suffers a myocardial infarction (MI or heart attack) and is
stabilized in the hospital. The next day his heart rate is less than 35 beats per
minute. Which one of the following agents might be administered to treat him?
a) atropine
b) succinylcholine
c) tubocararine
d) pilocarpine
e) pyridostigmine
B
A 9 year old boy is rushed to the Emergency Department wheezing with labored breathing and
falling blood pressure. His mother tells the physician that he was stung by a bee about 30
minutes ago. Which agent is given immediately to counteract the boy's condition?
a) dobutamine
b) epinephrine
c) clonidine
d) amphetamine
e) ephedrine
C. Increase neurotransmitter degradation, usually through treating the patient with Pralidoxime
An agricultural worker is brought to the Emergency Department after abrupt
onset of bowel and bladder incontinence and muscle weakness. He is given
oxygen and antidotal drug treatments. Which drug mechanism would increase
muscle strength in this patient?
a) blockade of muscarinic receptors
b) activation of nicotinic receptors
c) increased neurotransmitter degradation
d) induction of drug metabolizing enzymes
e) increased urinary excretion of weak acids
B. Dilation of the pupils, probably through cocaine use.
A man is arrested while using a substance that inhibits the neuronal catecholamine transporter.
Which signs would most likely be observed in this person?

a) excessive sweating
b) dilation of the pupils
c) involuntary muscle contractions
d) flushing of the skin
e) sedation
C. Alpha-1 adrenergic agonist would
A man with diabetic autonomic neuropathy complains of dizziness and fainting when arising
from bed in the morning. A drug activating which receptors would be most beneficial to this
patient?

a) a β1 adrenergic agonist
b) a β2 adrenergic agonist
c) an α1 adrenergic agonist
d) α2 adrenergic agonist
e) a non-selective β adrenergic agonist
A. Increased cyclic AMP levels
After being stung by a yellow jacket, a woman experiences urticaria, laryngeal
edema, difficult breathing and hypertension. She receives oxygen and administration of an
adrenergic agonist. Which action would lead to bronchodilation?

a) increased cyclic AMP levels


b) increased cyclic GMP levels
c) increased IP3 levels
d) calcium influx
e) sequestration of calcium
D.

D(L) = (Vd * Css)/F


D(L) = (500L * 2ng/mL)/1
D(L) = (500L * 2e-6mg/(1e-3 L))/1
D(L) = 1 mg/liter
Assume the following data for drug A, Vd = 10liters/kg, clearance =
0.1liters/kg/hour, the drug is 100% absorbed and does not undergo
biotransformation. The desired therapeutic concentration is 2ηg/ml. What
loading dose should be administered to a 50kg young woman?
a) 0.1mg/liter
b) 10μg/liter
c) 100/liter
d) 1mg/liter
e) 10mg/liter
D. Labetalol. In addition to being beta blockers, Labetolol and carvedilol are alpha-1 blockers.
You have a patient with essential hypertension that requires a drug that reduces both cardiac
output and peripheral resistance. A good choice would be:
a) metoprolol
b) phentolamine
c) timolol
d) labetalol
e) phenoxybenzamine
B.
In general induction of CYP450 enzymes involves increased gene transcription
leading to increased enzyme content. Induction of which of the following would
affect the largest number of drugs?
a) CYP 2E1
b) CYP 3A4
c) CYP 1A2
d) CYP 2C9
e) CYP 2D6
A
A patient is on "Drug A" for hypertension (the drug lowers blood pressure). Drug
A is a substrate of the CYP450 enzyme 3A4, and it is metabolized to an inactive
metabolite. Drug A has been working well for years and the patient's blood pressure is under
good control. The patient is then prescribed a new medicine for his heart, "Drug B". Drug B is an
inhibitor of 3A4. What will be the potential clinical consequence of this patient taking Drug B?

a) his blood pressure will decrease


b) his blood pressure will increase
c) the heart medicine will not work
d) he will have side effects related to the heart
B
As we age, lean body mass tends to decrease, fat tends to increase. Which
pharmacokinetic parameter do these changes most affect?
a) absorption
b) distribution
c) hepatic metabolism
d) renal excretion
B. Isoproterenol is a beta agonist, which would increase heart rate and decrease SVR.

Phenylephrine is an alpha-1 agonist, which would increase SVR.


Ephedrine is like longer acting epi, it would cause vasoconstriction.
Amphetamine would also not decrease SVR.
Dobutamine, a beta-1 agonist, would increase blood pressure.
The following graphs depict the effect of 3 drugs on systolic, diastolic and mean
arterial blood pressure as well as the effect on heart rate following the iv administration of drugs
X, Y and Z. If drug X is norepinephrine, and drug Y is epinephrine, drug Z is most likely:
a) phenylephrine
b) isoproterenol
c) ephedrine
d) amphetamine
e) dobutamine
Error! Filename not specified.
B. Metoprolol, a beta blocker.
the increase in heart rate produced by drug Z could be blocked by prior
administration of:
a) atropine
b) metoprolol
c) phentolamine
d) trimethaphan
e) reserpine
Error! Filename not specified.
B. Clonidine, an alpha-2 agonist.
The accompanying table shows cardiovascular changes induced by drug treatment in
experimental subjects. Which of the following drugs is most likely to cause the changes shown
in profile 4:

a) amphetamine
b) clonidine
c) epinephrine
d) isoproterenol
e) tyramine
Error! Filename not specified.
A. administration of bethanechol, which is a muscarinic agonist.
In the accompanying figure, changes in bronchomotor tone in experimental subjects are
depicted in terms of constriction (+) or relaxation (-). The first panel of the figure (control) shows
changes that were elicited by five different procedures (numbered 1 through 5), each of which
was performed alone. The remaining panels show changes that were elicited by the same five
procedures, each of which was performed after pretreatment with a different drug. Based on the
results, procedure 1 is most likely:

a) administration of bethanechol
b) administration of histamine
c) administration of isoproterenol
d) stimulation of sympathetic nerves
e) stimulation of or the vagus nerve
Error! Filename not specified.
C. Prazosin, which is an alpha-1 blocker, along with Tamsulosin and Doxasozin (Cardura)
The above schema shows the effect of norepinephrine (NE), Drug X and Drug Y at the
sympathetic neuroeffector junction. Based on the sequence of events that NE, Drug X and Drug
Y produce, Drug Y is most likely which of the following:

a) phentolamine
b) phenoxybenzamine
c) prazosin
d) propranolol
e) metoprolol
Error! Filename not specified.
B. New Drug Application (NDA)
Following phase 3 clinical trial, if a drug meets expectations, a drug company will make an
application to the FDA for permission to market the new drug. Such an application is called:

a) Notice of Claimed Investigational Exemption for a New Drug (IND)


b) New Drug Application (NDA)
c) Prescription Drug User Act (PDUA)
d) Research, Development and Marketing Application (RDMA)
e) Post Market Surveillance Application (PMSA)
A. propanolol, a beta blocker
The above drawing depicts the heart rate of a patient with ischemic heart disease measured by
telemetry while watching television. Measurements were begun 1 hour after receiving placebo
(upper line) or a dose of Drug X (lower line). Drug X is most likely which of the following:

a) propranolol
b) phentolamine
c) atropine
d) neostigmine
e) bethanechol
Error! Filename not specified.
D
Drug A has an EC50 of 10 nM and a maximum effect of 95%. Drug B has an EC50 of 75 nM
and a maximum effect of 100%. Drug C has an EC50 of 190 nM and a maximum effect of 34%.
Which of the following is correct?

a) drug B is the most potent drug


b) drug A is the most potent and has the highest efficacy
c) drug C has the lowest efficacy but is the most potent
d) drug A is the most potent drug
e) drug B is the least potent
B. (The spare receptor effect means there are many receptors in excess of what is required to
produce a response)
As a research scientist you have characterized a new beta blocker. You have determined from
binding/efficacy studies that this receptor does not have a "spare receptor" effect. In addition
your studies show the drug to be a simple, reversible competitive antagonist. Which of the
following statement best describes how the drug affects epinephrine function?

a) reduce the efficacy (Emax) (of epinephrine


b) shifts the EC50 of epinephrine to the right
c) not change the EC50 of epinephrine
d) shifts the EC50 of epinephrine to the left
e) increase the efficacy (Emax) of epinephrine
D
A patient is being treated for congestion with a drug that activates a G-protein coupled receptor.
The drug mechanism of action involves activation of phosphatidylinositol 4,5-bis phosphate
phospholipase C. The signaling pathway that is activated by this receptor results in which of the
following second messengers? 

a) cyclic AMP
b) stat translocation to the nucleus
c) cyclic GMP
d) inositol triphosphate
e) nitric oxide
D. Partial agonist, either Pindolol, Labetalol, or Celiprolol
When drug A is administered alone it results in an increase in heart rate by activating beta
adrenoreceptors. However, when you administer a highly effective beta stimulant, drug A
causes a dose dependent, reversible decrease in heart rate. Drug A is most likely?

a) a physiologic antagonist
b) a chemical antagonist
c) an irreversible antagonist
d) a partial agonist
e) a reversible antagonist
C
A drug company is testing two new high blood pressure drugs, drug A and drug B. Drug A and
drug B have half-lives of 4 hours and 8 hours, respectively. Both drugs exhibit the same Vd. You
administer 20mg/kg of each drug. Which of the following statements is most correct?

a) the steady state concentration of drugs A and B are equal


b) steady state concentration of drug A is greater than that of drug B
c) the clearance of drug A is greater than drug B
d) the times to reach steady state are equal
e) the elimination rate of drug A is less than drug B
B??
A drug company has decided to put a second generation drug, drug B on the market. Drug A is
a weak base. Drug B also a weak base has been modified when compared with the first
generation drug A Drug B's absorption from the small intestine (pH = 7) when compared to the
first generation drug A:

a) decreases as the partition coefficient of drug B increases compared with


drug A
b) increases as the pK of drug B decreases compared with drug A
c) decreases as the diffusion coefficient of drug B increases compared with
drug A
d) does not change as the pK of drug B increases compared with drug A
e) increases as the molecular weight of drug B increases compared with
drug A
E
A patient is administered i.v. 90mg of a drug. Blood samples are collected and the following
blood concentrations are recorded (see table below). Which of the following statements is most
correct?

a) the elimination rate is constant, first order


b) the elimination rate is constant, directly proportional to drug concentration
c) the half-life of the drug is 3 hours
d) the elimination rate is zero order, directly dependent on concentration
e) the elimination rate is constant independent of drug concentration
Error! Filename not specified.
D.
Use:
Dm = (Css Cl Tm)/F
A patient (70kg) is orally taking a beta blocker. The clearance of the drug is 50liters/day, the
volume of distribution is 0.5liters/kg and the bioavailability is 0.5, the desired therapeutic
concentration is 10μg/liter. To attain the therapeutic concentration the following dose should be
administered at what amount 4 times a day?

a) 2.5mg
b) 4mg
c) 4μg
d) 250μg
e) 1000μg
B
The patient's hematocrit is 0.4. The concentration of the drug in the plasma is 10μg/liter and the
concentration in the pelleted cells is 1μg/liter. Calculate the concentration of the drug in blood.
Choose the answer that is closest to the
calculated value.
a) 4.2μg/liter
b) 6.3μg/liter
c) 8.4μg/liter
d) 10μg/liter
e) 9.4μg/liter
D. Reserpine would deplete catecholamines, and thus prevent the effects of amphetamines,
which help with catecholamine release.
Amphetamine like drugs (dextro-amphetamine, methamphetamine) are known to have marked
CNS activity including wakefulness, alertness, decreased feeling of fatigue, self confidence,
elevation of mood as well as peripheral effects such as an increase in blood pressure and
myocardial stimulation. Pretreatment of which of the following drugs would prevent these effects
of dextro-amphetamine.

a) nadolol
b) atropine
c) clonidine
d) reserpine
e) terbutaline
B.
Use:
D(L) = (Vd * Css)/F
With Css = 10 ug/L, which is total blood concentration:
Plasma concentration = 9ug/L
Bound to serum proteins = 1ug/L
A 70kg male is taking drug A, a vitamin K inhibitor. The drug is 50% absorbed within minutes of
oral administration. None of the drug undergoes biotransformation. The volume of distribution =
10 liters. Ten percent of the drug binds to serum proteins. What dose of drug should be given to
attain an interstitial concentration of 9ug/liter of drug, if the drug is administered once a
day?

a) 0.1mg
b) 0.2mg
c) 0.3mg
d) 0.4mg
e) 0.8mg
C
Why should the use of amitriptyline be avoided in older persons?

a) It has a long half-life


b) It has a short half-life
c) It has strong anticholinergic properties
d) It lacks effectiveness compared to other drugs in its class
e) It has potential for several drug interactions because of it metabolism by
the P450 system in the liver
A. Milliseconds is key word here. PLC activation takes a little bit longer
You are studying the effect of a drug in cell culture and find that it stimulates a marked increase
in intracellular calcium. The increase in calcium occurs within milliseconds of addition of drug.
The increase in intracellular calcium is likely a
result of:

a) the drug binding directly to a ligand gated calcium channel


b) the drug binding to a receptor that activates a calcium channel via PKA
c) the binding to a G-protein that activates a calcium channel via release of
inositol triphosphate
d) the drug binding to a nuclear receptor
e) none of the above
C.

Physiological antagonism refers to an interaction which results from one drug antagonizing the
response of another drug following stimulation of a different receptor.

This is a type of nonreceptor antagonism, along with chemical antagonism, in which the
antagonist sequesters agonist and prevents its activity.

There is also receptor antagonism, which may be competitive or noncompetitive.


Two drugs are being tested. Drug binding studies have determined that the drugs
act on different receptors. It has also been determined that when the drugs are
administered the two drugs have opposing actions in the body which tend to
cancel one another. This is known as:

a) chemical antagonism
b) non-competitive antagonism
c) physiologic antagonism
d) pseudo-irreversible antagonism
e) competitive antagonism
E
The figure shows the plasma concentration of an antibiotic as a function of time obtained after
an i.v. injection of 7mg.

If this drug is given as a constant i.v. drip it will attain its therapeutic concentration at what time
after the start of the administration.
a) ½ hr
b) 1 hr
c) 2 hrs
d) 3 hrs
e) 5 hrs
Error! Filename not specified.
E
Vd = dose/Cp(0)
The figure shows the plasma concentration of an antibiotic as a function of time obtained after
an i.v. injection of 7mg.

Calculate the volume of distribution of the drug.

a) 0.2 liters
b) 0.3 liters
c) 0.35 liters
d) 3 liters
e) 3.5 liters
Error! Filename not specified.
B
Margin of safety = TD1/ED99
Therapeutic index = TD50/ED50
Drug A was studied in a large group of patients and the percentages of the group
showing a specific therapeutic effect was determined and toxic effect was
determined (see the table below). Calculate the Margin of Safety (Certain Safety
Factor) of drug A.

a) 0.33
b) 1
c) 10
d) 3.3
e) 33
Error! Filename not specified.
E. Internalization of receptor proteins
During the first 2 weeks of therapy with a beta 2 adrenoceptor agonist, a 50 year old man with
asthma complains that his inhaler is gradually losing effectiveness. Which
mechanism is most likely responsible for this effect?
a) autoinduction of drug metabolizing enzymes
b) formation of antibodies to the drug
c) exacerbation of his disease
d) depletion of adenosine triphosphate
e) internalization of receptor proteins
C
For drug X, the volume of distribution is normally 35L, and 80% of the drug is bound to plasma
proteins. In patients with hypoalbuminemia, plasma protein binding is reduced to
60%. You now measure the amount of drug in the plasma volume and find:

a) the amount of drug in the plasma volume increases


b) the amount of drug in the plasma volume does not change
c) the amount of drug in the plasma volume decreases
d) the amount of drug in the interstitial volume and tissues does not change
e) the amount of drug in the interstitial volume decreases
B, botulism toxin.
A 33-year old woman with intractable torticollis (persistent contraction of the neck
muscles causing the head to be twisted to an abnormal position) obtains relief from a
localized injection of a bacterial toxin. Which step is most likely inhibited by this drug?
a) transport of the neurotransmitter precursor into neurons
b) fusion of transmitter vesicles and cell membrane
c) neurotransmitter storage in neuronal vesicles
d) cytoplasmic neurotransmitter synthesis
e) synaptic neurotransmitter degradation
D. 

Concentration of unbound drug in plasma = concentration of drug in interstitial space. Blood +


interstitial space = extracellular space. So total interstitial space volume = 10 - 3 = 7.
Total concentration of unbound drug = (1-percent bound to serum proteins)(blood
concentration) = (.4)3 = 1.2.
1.2*7 = 8.4 mg
Drug A is completely ionized at a pH of greater than 7. The pH of the blood and plasma are 7.4.
You have determined that 60% of the drug binds to serum proteins. The drug concentration in
the blood is 3mg/liter. The extracellular volume is 10 liters and the plasma volume is 3 liters.
Calculate the amount of drug in the interstitial volume that is
available for binding to its targeted receptor.
a) 1.8mg
b) 5.4mg
c) 5mg
d) 8.4mg
e) 13.3mg
C.

Reserpine inhibits VMAT depleting vesicles of catecholamines

Guanethidine is transported across cell membrane by same NE reuptake transporters, and


eventually depletes vesicles of NE.

Metatyrosine (alpha-methyl-p-tyrosine) is an inhibitor of tyr hydroxylase.

Alpha-methyldopa inhibits DOPA Decarb.

Clonidine is an alpha-2 agonist.


A 51 year old man with daily episodes of severe hypertension usually lasting 1-3 hours receives
a drug that inhibits neuronal utilization of tyrosine. The drug that was given to this patient is
most likely:

a) reserpine
b) guanethidine
c) metatyrosine (alpha methylparatyrosine)
d) alpha-methyldopa
e) clonidine
C. Phospholipase C

Eccrine sweat glands have mostly M3 receptors. Muscarinic receptors 1,3,5 are excitatory, and
activate PLC through Gq/11, leading to increases in DAG and IP3, and sometimes PLA2 and
arachidonic acid release. Muscarinic receptors 2 and 4 have Gi/0 subunits that lead to inhibition
of adenylate cyclase and decrease in cAMP.
A one week old infant with failure to thrive was evaluated for aptic fibrosis by collecting sweat
via dermal iontophoresis of a drug that evokes eccrine sweat gland secretion. Which enzyme is
activated during the signaling cascade responsible for stimulating
sweat gland secretion?

a) cholinesterase
b) adenylyl cyclase
c) phospholipase C
d) phosphodiesterase
e) choline acetyl transferase
E. Release of calcium from sarcoplasmic reticulum.

A good way to treat orthostatic hypotension is to increase blood pressure, and a good way to
increase blood pressure is to activate alpha-1 receptors in vasculature in order to raise SVR.
Alpha-1 receptors activate through a Gq/11 subunit to activate PLC, increasing IP3 and DAG,
and increasing intracellular Ca2+.
A 55-year-old woman with diabetic autonomic neuropathy complains of feeling lightheaded and
dizzy whenever she gets out of bed. Examination reveals that has blood
pressure falls abruptly upon arising from the supine position and she is placed on drug therapy
for the condition. Which action is most likely responsible for the therapeutic
effect of the patients drug therapy:

a) stimulation of adenylyl cyclase and increased cyclic AMP


b) inhibition of adenylyl cyclase and decreased cyclic AMP
c) increased potassium conductance and membrane hyperpolarization
d) decreased potassium conductance and membrane depolarization
e) release of calcium from the sarcoplasmic reticulum
B. EpiPen
After receiving an intramuscular injection of penicillin, a woman experiences swelling of the
throat and tongue, hives and labored breathing. She is given an intramuscular injection of an
adrenoceptor agonist. The adrenoceptor agonist is most likely:

a) clonidine
b) epinephrine
c) isoproterenol
d) phenylephrine
e) dobutamine
B. Timolol. Used to decrease humor flow.

Pilocarpine is a muscarinic agonist which helps with drainage in wide angle glaucoma.
A man is found to have an intraocular pressure of 24 mmHg (normal 10-20) but no evidence of
optic nerve damage. He is placed on an antagonist of G-protein coupled receptor in order to
lower his intraocular pressure. Which drug was most likely used:
a) pilocarpine
b) timolol
c) clonidine
d) isofluorphate (DFP)
e) epinephrine
B.

Most likely he was put on an alpha-1 antagonist such as Tamsulosin. Alpha-1 antagonist can
also decrease vascular tone and thus increase risk for postural hypotension.
A man complains of needing to urinate frequently in the daytime and at night and of difficulty of
maintaining urine outflow during micturation. Examination reveals prostatic enlargement. His
prostate specific antigen (PSA) is normal and he is placed on a drug to relax prostate and
trigone smooth muscle. Which side effects may be likely to result from taking this medication?

a) bradycardia
b) postural hypotension
c) insomnia
d) priapism
e) constipation
C. Tamulosin is a alpha-1 antagonist used in the treatment of BPH, and it also blocks the
receptor responsible for trigone and sphincter contraction.
A man complains of needing to urinate frequently in the daytime and at night and of difficulty of
maintaining urine outflow during micturition. Examination reveals prostatic enlargement. His
prostate specific antigen (PSA) is normal and he is placed on a drug to
relax prostate and trigone smooth muscle. The drug administered to this patient is most likely:
a) tolterodine
b) ipratropium
c) tamulosin
d) atropine
e) propranolol
B. Stomach
Which organ is least important when considering changing pharmacokinetics and normal aging
(that is, not due to disease)?

a) Brain
b) Stomach
c) Liver
d) Kidneys
e) Heart
D. Clonidine, which is an alpha-2 agonist that dampens sympathetic response.

Phentolamine is a alpha blocker that doesn't affect the heart directly, Metoprolol is a
beta1>beta2 blocker, Propanolol is a beta blocker, Phenoxybenzamine is an alpha antagonist.
A woman presents to an afterhours medical clinic with a blood pressure of 178/98 (normal range
120-129/80-89). She has been taking antihypertensive medication but ran out of medicine
several days ago. The physician orders a single dose of a drug that
reduces sympathetic tone to the heart and circulation in order to lower her blood
pressure. The drug prescribed is most likely:
a) phentolamine
b) metoprolol
c) propranolol
d) clonidine
e) phenoxybenzamine
C. Dobutamine is a beta-1 agonist that leads to Gs activation, rise in cAMP. Increases inotropy
and HR, with net effect raise in cardiac contractility and CO. Tends to mildly decreas SVR due
to counterbalancing beta-2 and alpha 1 effects.

DESPITE raise in HR, dobutamine can actually decrease myocardial oxygen demand by raising
inotropy and lowering SVR.
Used in the acute management of decompensated CHF. Arrhythmias and possible
Hypotension?
A 72 year old woman is treated for acute heart failure with an infusion of dobutamine. The
duration of dobutamine infusion should be limited to several hours each day
because prolonged infusion may lead to drug tolerance. The principal receptor being activated
by dobutamine is:

a) α1
b) α2
c) ß1
d) ß2
e) ß3
B
A 29 year old woman with myasthenia gravis is being treated with prednisone and
pyridostigmine. She complains to her physician of increasing diplopia and facial weakness. If an
intravenous dose of edrophonium improves her muscle weakness in this patient indicates:

a) the diagnosis is incorrect


b) the dose of pyridostigmine should be increased
c) the dose of pyridostigmine should be reduced
d) atropine should be added to the regimen
e) prednisone should be discontinued
C. Prazosin is a alpha-1 blocker, Propanolol is a beta blocker, Clonidine is a alpha-2 agonist,
Dobutamine is a beta-1 agonist, Phenoxybenzamine is an alpha blocker.
A 33 year old man experiences the episodic attacks of severe hypertension caused by an
adrenal medullary tumor that secretes copious amounts of epinephrine and
norepinephrine and he is scheduled for surgery to remove the tumor. Which of the following
agents would shift the dose response curve of norepinephrine to the right without decreasing
the maximal effect:

a) propranolol
b) clonidine
c) prazosin
d) dobutamine
e) phenoxybenzamine
C. Atropine, which is a muscarinic antagonist. Patient's symptoms are due to muscarinic
overactivity, and this would bring it back in line.

Tamulosin is an alpha-1 blocker, Phentolamine is an alpha-blocker ( along with


phenoxybenzamine), Physostigmine is a AChEI, vecuronium is a nondepolarizing inhibitor of
nicotinic receptor at neuromuscular junction.
A patient is experiencing spasm of accommodation (cyclospasm), miosis, increased gut motility,
excessive salivation and increased sweating following drug administration. An
agent that could reduce these symptoms is:
a) phentolamine
b) tamulosin
c) atropine
d) physostigmine
e) vecuronium
C. Cocaine, which blocks reuptake of NE and would thus lead to exaggerated response.

Propanolol is a beta blocker, prazosin is a alpha-1 blocker, isoproterenol is a beta agonist.


In the following figure, mean arterial blood pressure is being continuously measured in an
anesthetized mammal. Continuous infusion of drugs 1, 2 and 3 are started at the times indicated
by the arrows. Epinephrine (E) is injected at each time indicated by E.

Compound 1 is most likely:


a) propranolol
b) prazosin
c) cocaine
d) acetylcholine
e) isoproterenol
Error! Filename not specified.
A. Prazosin, which is an alpha-1 blocker. If alpha-1 is blocked, then beta-2 will predominate in
vasculative, leading to a substantial decrease in SVR.

Phenylephrine is an alpha-1 agonist. This is in contrast to Prazosin.


In the following figure, mean arterial blood pressure is being continuously measured in an
anesthetized mammal. Continuous infusion of drugs 1, 2 and 3 are started at the times indicated
by the arrows. Epinephrine (E) is injected at each time indicated by E.

Compound 2 is most likely:


a) prazosin
b) atropine
c) propranolol
d) acetylcholine
e) phenylephrine
Error! Filename not specified.
C. Propanolol
In the following figure, mean arterial blood pressure is being continuously measured in an
anesthetized mammal. Continuous infusion of drugs 1, 2 and 3 are started at the times indicated
by the arrows. Epinephrine (E) is injected at each time indicated by E.

Compound 3 is most likely:


a) acetylcholine
b) atropine
c) propranolol
d) amphetamine
e) ephedrine
Error! Filename not specified.
C. Atropine
A 55 year old man is diagnosed with Sjogren's syndrome which is characterized by
chronic dry mouth due to under secretion of saliva. He is placed on medication Drug X to
remedy the situation. Which of the following would antagonize Drug X?

a) pyridostigmine
b) propranolol
c) atropine
d) prazosin
e) metoprolol
C. Pindolol.

Pindolol, along with Labetalol, and Celiprolol have what is called partial agonist activity.
A drug is administered to a patient that increases heart rate in the absence of norepinephrine
but blocks norepinephrine induced positive inotropic and chronotropic
effects when the two are present together. It is most likely:

a) nadolol
b) metoprolol
c) pindolol
d) timolol
e) labetalol
C. Prazosin, which is an alpha-1 blocker
Administration of a high dose of epinephrine to a patient produced a paradoxically decrease in
mean arterial pressure. Which of the following drugs may the patient have previously taken to
account for this unexpected effect?

a) cocaine
b) atropine
c) prazosin
d) reserpine
e) propranolol
C. Phase III

Phase I and II are generally smaller. Phase I is to assess safety, bio effects, metabolism,
kinetics, drug itneractions, usually done by clinical pharmacologists (Is it safe?).
Phase II is through selected patients to assess therapeutic efficacy, dose range, kinetics,
metabolism (Does it work?).
Phase III is a large sample used to assess safety, efficcacy, esp. compared to other drugs,
placebos.
Phase IV is post drug approval, postmarket surveillance.
Phase V, doesn't exist.
A drug is being developed that will selectively bind and reverse the effects of skeletal muscle
relaxants. This new medication will not possess the unwanted muscarinic effects of neostigmine
which is currently used for that purpose. The drug appears to be effective. However, in a large
study group/trial prior to the release of the drug on the
market, it was discovered that the medication might produce allergic reactions at an
unacceptable high rate. Release of the drug was held back by the FDA. At what phase
of drug development was this possible allergic side effect discovered?

a) Phase I
b) Phase II
c) Phase III
d) Phase IV
e) Phase V
D
You are administering a drug orally and it has a narrow therapeutic window. You want to
decrease the plasma concentration fluctuations. Which of the following actions would
decrease the fluctuations without altering the average steady state drug level for a person on an
intermittent dose regimen?

a) double the size of the dose and keep dose interval the same
b) double the dose interval and keep the size of the dose the same
c) double the dose interval and the decrease the size of the dose by 1/2
d) decrease the dose interval by 1/2 and decrease the size of the dose by 1/2
e) decrease the dose interval by one half and increase the dose by factor of 2
E. Brain. Highly ionized and quarternary molecular structure is associated with low penetrance
of BBB.
A 14 year old boy is brought by his mother to his primary care physician after development of
fever, nausea, headache and diffuse muscle pain. He has a macular rash that started on his
hands and feet and has spread toward his chest and thighs. His physician prescribes an
antibiotic that has a highly ionized, quaternary molecular structure. Which organ is likely to
exhibit the lowest concentration of this drug:

a) kidney
b ) liver
c) spleen
d) heart
e) brain
B. Oxidation.
Oxidation is rarely a part of phase II reactions. Phase I reactions usually include oxidation,
reduction, and hydrolysis.

Phase II reactions usually include sulfate conjugation, glucuronidation, methylation, acetylation,


mercapturic acid formation.
A one year old baby boy with persistent jaundice is evaluated by a pediatric gastroenterologist,
who determines that the patient has defects in several Phase II metabolic pathways and
cautions against using medications dependent upon these reactions. However, the physician
states that the baby's phase I metabolism is probably
intact. Which of the following pathways of drug metabolism is most likely spared?

a) sulfation
b) oxidation
c) methylation
d) glucuronidation
e) acetylation
E
A 42 year old man with chronic low back pain attributed to a compressed nerve is scheduled to
undergo a lumbar laminectomy in the prone position. Prior to the procedure an anesthesiologist
administers a muscarinic antagonist to reduce the patient's salivation. Which of the following
additional side effects of the medication would be most likely to occur?

a) bradycardia
b) miosis
c) hyperhidrosis
d) diarrhea
e) urinary retention
A. Acetylcholine. 

Vasodilation due to muscarinic receptors on peripheral sympathetic nerve terminals that block
the release of NE thus reducing sympathetic vascular tone.

For vasoconstriction, think baroreceptors?


In experimental studies when anesthetized subjects are treated intravenously with a large or
small dose of drug X alone, they exhibit vasodilation. If they are treated first with atropine and
then given a small dose of drug X (1μg/kg), the vasodilation is blocked. However if given a large
dose of drug X (100 μg/kg) they exhibit a vasoconstrictive response and this response is
potentiated by physostigmine. What agent is drug X likely to be?

a) acetylcholine
b) epinephrine
c) histamine
d) isoproterenol
e) norepinephrine
E
After ingestion of a massive dose of cocaine an 18 year old man participates in a game of
soccer and suffers a fatal myocardial infarction. What autonomic mechanism was blocked by
cocaine to contribute to cardiac death in this individual?

a) β-adrenergic receptors
b) incorporation of norepinephrine into storage vesicles
c) release of norepinephrine from sympathetic nerve terminals
d) synthesis of norepinephrine in sympathetic nerves
e) uptake of catecholamines into sympathetic nerve terminals
E. Propranolol
In experiments when anesthetized subjects are treated intravenously with an agonist (drug X)
they have an increase in their blood glucose and in the force ventricular contraction, and a
decrease in peripheral resistance. Pretreatment of the subjects with
an antagonist (drug Y) almost completely blocks all of these effects. 

Which agent is drug Y likely to be?

a) terbutaline
b) atropine
c) guanethidine
d) phentolamine
e) propranolol
C. Isoproterenol, which is a beta agonist
In experiments when anesthetized subjects are treated intravenously with an agonist (drug X)
they have an increase in their blood glucose and in the force ventricular contraction, and a
decrease in peripheral resistance. Pretreatment of the subjects with an antagonist (drug Y)
almost completely blocks all of these effects. 

Drug X is:

a) terbutaline
b) epinephrine
c) isoproterenol
d) bethanechol
e) phenylephrine
B. Bethanechol is a muscarinic agonist, muscarinic receptors --> bronchoconstriction. Timolol
will also cause bronchoconstriction but through inhibition of beta-2 receptors that cause
bronchodilation.
Atropine is a muscarinic antagonist, isoproterenol is a beta-1,2 agonist, and phentolaine is an
alpha blocker. These would either shift to bronchodilation or have little effect.
A 35-year old woman has severe bronchial asthma. What agent would most likely cause a
severe bronchospasm reaction in this patient?

a) atropine
b) bethanechol
c) timolol
d) isoproterenol
e) phentolamine
E. Norepinephrine
In experimental studies when anesthetized subjects are treated intravenously with drug X, they
show a marked increase in both diastolic and systolic blood pressure and a
slowing of the heart rate. However, if they are pretreated with prazosin, they show a modest
increase in the systolic blood pressure and heart rate. What agent is drug X?

a) acetylcholine
b) epinephrine
c) histamine
d) isoproterenol
e) norepinephrine
A. Albuterol, which is a beta-2 agonist.
A 13 year-old girl has severe, frequently recurring asthmatic attacks. During the attacks, she
becomes cyanotic, dyspneic and agitated. What drug is expected to provide immediate relief
with the fewest adverse effects in controlling asthmatic attacks in this patient?

a) albuterol
b) atropine
c) ephedrine
d) epinephrine
e) phenylephrine
C. Phenylephrine is an alpha-1 agonist. Mydriasis is controlled by alpha-1 receptors in iris.

There would be cardiac deceleration because alpha-1 receptors would increase BP.

Relaxation of bronchial smooth muscle is controlled by beta-2.

Relaxation of uterine smooth muscle is through beta-2, constriction is through alpha-1.


A 34 year-old woman takes a product that contains acetaminophen (analgesic),
chlorpheniramine (antihistamine) and phenylephrine to relieve symptoms associated with the
common cold including itchy and watery eyes, runny nose and wheezing. What adverse effects
is potentially caused by the phenylephrine part of the regimen?

a) cardiac acceleration produced by reflex mechanisms


b) decreased production of aqueous humor in the eye
c) mydriasis
d) relaxation of bronchial smooth muscle
e) relaxation of uterine smooth muscle
D
Tachyphylaxis occurs when an indirectly acting sympathomimetic amine, such as
amphetamine, is administered repeatedly over a short time span. Which statement best
described the mechanism of tachyphylaxis produced when amphetamine is administered
repeatedly over a short time period.

a) blockade of nerve conduction in adrenergic nerve terminals


b) blockade of receptors
c) compensatory parasympathetic response
d) decreased stores of available transmitters
e) increased destruction of the drug
D
The process of investigating new drugs in clinical studies is strictly regulated to ensure the
safety of human subjects. Clinical studies are divided into several phases. What do
Phase II clinical studies typically involve?

a) collecting and analyzing liver toxicity from phase I


b) determining pharmacokinetic properties of these new drugs when used in healthy
volunteers
c) establishing that the new drug is non-toxic in the proposed dosage range
d) evaluating the new drug in a small group of patients with the targeted disease
e) performing a large scale evaluation of the adverse effects of the new drug
A. Brimonidine, which is an alpha-2 adrenergic agonist, decreases synthesis of aqueous humor
through Gi --> decrease in cAMP
A woman requires a drug to lower intraocular pressure after cataract surgery. Which mechanism
would most likely lead to the desired effect?

a) inhibition of adenylyl cyclase


b) stimulation of adenylyl cyclase
c) activation of phospholipase C
d) inhibition of phospholipase C
d) release of calcium from sacroplasmic reticulum
B. Rapid heart rate. B2 --> decrease in SVR --> tachycardic baroflexive response
A child with asthma is being treated with an adrenergic agonist to prevent
bronchospasm. Which side effect is typically associated with this drug?

a) sedation
b) rapid heart rate
c) muscle weakness
d) high blood pressure
e) blurred vision
A
After being stung by a bee, a woman experiences urticaria, laryngeal edema, difficulty breathing
and hypotension. She receives oxygen and the administration of an adrenergic agonist. Which
action would lead to bronchodilation?

a) increased cAMP levels


b) increased cyclic guanosine monophosphate (cGMP) levels
c) increased IP3 levels
d) calcium influx
e) sequestration of calcium
B
Which of the following produces transient muscle fasiculations followed by muscle paralysis and
is not reversed by neostigmine?

a) ipratropium
b) succinylcholine
c) vecuronium
d) scopolamine
e) d-tubocurarine
A (because it is metabolized to an active metabolite)
A patient is on "Drug A" which is a type of blood thinner used to "keep his stents in his coronary
arteries open." Drug A is a substrate of the CYP450 enzyme 2C19, and it is metabolized to an
active metabolite. The patient decides to take a second drug, "Drug B" for acid reflux. Drug B is
an inhibitor of 2C19. What will be the potential clinical consequence of this patient taking Drug
B?

a) He could have a heart attack.


b) He could have side effects related to the acid reflux medicine.
c) His blood could become "too thin."
d) His liver could suffer temporary damage.
e) The reflux medicine may not work.
C
In the absence of other drugs, you administer a drug that produces a small increase in heart
rate by activating β-adrenergic receptors. In the presence of epinephrine, however, addition of
the drug results in a dose-dependent reversible decrease in the ability of epinephrine to
increase heart rate. The drug is:

a) a non-competitive antagonist
b) a irreversible partial agonist
c) a partial agonist
d) a chemical antagonist
e) a physiologic antagonist
a) prolonged occupation of nicotinic receptors by acetylcholine
A 27-year old woman being treated with pyridostigmine complains of blurred double vision and
facial drooping 2 weeks after her last appointment with her doctor. After injection of a diagnostic
agent intravenously, her grip strength declines rapidly. Which
mechanism is most likely responsible for this loss of muscle tone?

a) prolonged occupation of nicotinic receptors by acetylcholine


b) transient depolarization of skeletal muscle
c) competitive blockade of nicotinic acetylcholine receptors
d) up regulation of nicotinic receptors in skeletal muscle
e) blockade of presynaptic acetylcholine receptors
a) elimination of the drug does not change as the plasma concentration increases
The drug that you have prescribed for your patient is eliminated by a carrier mechanism.
When the drug is at saturation:

a) elimination of the drug does not change as the plasma concentration increases
b) elimination of the drug increases as plasma concentration increases
c) the half-life of the drug increases as the plasma concentration increases
d) the drug will reach steady state faster than 5 half-lives
e) clearance of the drug increases as the plasma concentration increases
E
You, the physician, are taking care of a 74 year old woman. She is taking 3 medicines for her
high blood pressure, 2 medicines for her arthritis, 1 medicine for her insomnia, 2 medicines for
her diabetes, 2 laxatives, and 1 bladder medicine. What is the best thing
you can do to decrease her risk of adverse drug events (ADEs)?

a) Check her liver function and dose the medications accordingly


b) Check her renal function and dose the medications accordingly
c) Give her a medication planner (an organizer) to help her organize her pills
d) Stop any "dangerous" drugs based on her age
e) Try to decrease the number of medications
a) α1 adrenoceptors
A one week old infant in the neonatal intensive care unit requires intravenous infusion of an
agonist drug to increase cardiac output and counterbalance bronchoconstriction. After the
infusion is begun accidental extravasation of the drug solution results in tissue ischemia and
pain at the site of injection. Blood flow could be restored by infiltration of a drug that blocks
which receptor?
a) α1 adrenoceptors
b) α2 adrenoceptors
c) ß1 adrenoceptors
d) ß2 adrenoceptors
e) α1 and ß1 adrenoceptors
c) cyclobenzaprine (muscle relaxant), most muscle relaxants and antispasmodics are poorly
tolerated by elderly patients
A robust appearing 88 year old man (still plays tennis and golf), is admitted to the hospital
because he fell off his bike and sustained a fracture of his right femur. The
orthopedic surgeon has ordered the following medications to be used on an as needed basis:
morphine (narcotic), ibuprofen (non-steroidal anti-inflammatory), cyclobenzaprine
(muscle relaxant), acetaminophen (Tylenol), and acetaminophen+codeine (Tylenol #3).

Which of these is potentially inappropriate to use in an older person?


a) morphine (narcotic)
b) ibuprofen (non-steroidal anti-inflammatory)
c) cyclobenzaprine (muscle relaxant)
d) acetaminophen (Tylenol)
e) acetaminophen+codeine (Tylenol #3)
B. Tolterodine (Detrol) is a antimuscarinic drug that is used to treat urinary incontinence.

Bethanechol is a muscarinic agonist that would increase urinary urgency.

Atropine is a muscarinic antagonist, but can have many side effects.

Ipratropium is also a muscarinic antagonist, but it is poorly absorbed, and usually just inhaled to
treat COPD and asthma.

Neostigmine is a AChEI, not suitable.


A 57 year old woman complains of urinary frequency, urgency and bladder incontinence when
she reaches to get something out of the back seat of her car. A urine sample is negative for
glucose, protein, bacteria and blood cells. Which agent is most likely to relieve her symptoms
with minimal side effects?

a) atropine
b) tolterodine
c) ipratropium
d) neostigmine
e) bethanechol
C
The diagram above shows concentration response curves of drug A, in the presence of
increasing concentrations of an interacting drug X. Curve A is drug A alone, curves B - E are in
the presence of increasing concentrations of drug X. From the data it can be deduced that:

a) Drug X is acting as a physiological antagonist of drug A


b) Drug X has a higher affinity for its receptor than drug A
c) Drug X acts as a irreversible inhibitor of drug A
d) At the highest concentration tested (curve E) drug X acts as an partial agonist
e) Drug X is acting as chemical antagonist of drug A
Error! Filename not specified.
A
You are carrying out studies of a new formulation of a drug that will be taken orally..
You administered the drug I.V. at 1mg and orally at 10mg. The AUCoral= 1mg and the AUGi.v.
= 1mg. Calculate the bioavailability for the drug.
a) 0.1
b) 0.2
c) 0.5
d) 0.75
e) 1
C
A 41 year old woman with xerostomia (dry mouth) secondary to tobacco-related throat cancer
therapy is placed on a drug that increases intracellular calcium levels via activation of
phospholipase C. Which signal transduction mechanism participates in causing this effect?

a) conversion of adenosine triphosphate to cyclic AMP


b) dissociation of heat-shock protein and receptor dimerization
c) binding of guanosine triphosphate to a G protein
d) sodium influx
e) activation of nuclear receptors
C
You were treating a patient in the hospital by giving the drug i.v. The patient has been released
but needs to continue the medication. As his physician you have put the patient
on an oral formulation of the drug. Due to the nature of the gut membranes, drugs with
the following properties are more likely to cross most membrane barriers:
a) ionized and lipophilic
b) ionized and hydrophilic
c) nonionized and lipophilic
d) Nonionized and hydrophilic
B
Decreasing the pH of the intestinal contents is most likely to increase the rate of
absorption from the intestine of a drug that is a:
a) neutral organic drug
b) weak acid
c) weak base
d) a drug that is completely ionized before or after the change in pH
e) strong base
D

Cl = k(e) * Vd
with k(e) = 0.7/t(1/2).

T(1/2) = (0.7Vd)/Cl = (0.730)/3 = 7 hours


A volunteer will receive a new drug in a phase I clinical trial. The clearance and volume of
distribution of the drug are 3liters/hour and 30liters, respectively. The half-life of the drug is.

a) Less than an hour


b) 1 hour
c) 5 hours
d) 7 hours
e) More than 10 hours
C

Dm = (Css Cl Tm)/F
Cl = Vd*Ke = 4.6 L/hr
Vd = 2 L/kg * 80 kg = 160 L
Ke = 0.7 / t(1/2)
Dm = (10004.6/0.7)Tm
And the way it comes out is 80mg/12hours
Your patient has been diagnosed with a urinary tract infection and needs to be given a dosing
regimen of an antibiotic. Calculate a suitable dosage regimen for an 80 kg patient
who is to be given an oral fluoroquinolone. The therapeutic plasma concentration of the drug is
1.0μg/ml. The volume of distribution for this drug is 2 liters/kg and the half-life is 24 hours. The
oral bioavailability is 70%.

a) 50mg once a day


b) 80mg every 8 hours
c) 80mg every 12 hours
d) 160mg every 8 hours
e) 160mg every 12 hours
B
A decrease in heart rate and A-V conduction elicited by electrical stimulation of parasympathetic
nerves can be blocked by prior treatment with which of the following:
a) neostigmine
b) scopolamine
c) propranolol
d) clonidine
e) prazosin
D
A patient on digoxin therapy reports signs of toxicity, nausea with occasional vomiting.
The plasma digoxin level in this patient is 4 ng/mL. The half-life for elimination of digoxin is 2
days. How long should you withhold digoxin in order to reach a safer yet probably
therapeutic level of 1.0ng/mL?
a) 12 hours
b) one day
c) two days
d) four days
e) six days
C

Dl = (Vd * Css)/F
Dl = (20 L * 30mg/L)
Dl = 600 mg
The therapeutic range of a drug is 20-40mg/liter. The half-life of the drug is 2days,
clearance is 7liters/day and the volume of distribution is equal to 20liters. Calculate the
loading dose that is required to attain the therapeutic concentration.
a) 20mg
b) 200mg
c) 600mg
d) 800mg
e) 1600mg
D
You have just developed a new agonist, drug X, for a nicotinic ion channel. You plot the effects
of several drugs including drug X. In the graph below, curve X represents a concentration-
response of drug X, which is a full agonist (Emax= 100%). In comparison
to curve X, which curve represents the concentration-response of drug X in the presence
of a competitive reversible antagonist?
a) Curve A
b) Curve B
c) Curve C
d) Curve D
e) Curve X
Error! Filename not specified.
D
A pharmaceutical company has developed a new antagonist against beta 1 adrenergic
receptors. You administer the antagonist and observe a decreased in the activation of Gs. You
would expect to observe:
a) increase in calcium
b) increase in inositol triphosphate
c) increase in DAG
d) decrease in cAMP
e) decrease in inositol triphosphate
A) less than 1%
Drug A is a base with a pK of 9.0. The drug is eliminated by the kidneys exclusively via passive
diffusion. The pH of the plasma is 7. Calculate the % of drug in the plasma that is unionized.

a) less than 1%
b) less than 5% but greater than 1%
c) less than 10% but greater than 5%
d) less than 20% but greater than 10%
e) exactly 1%
E. Cocaine.

Cocaine is a NE uptake inhibitor, that will potentiate NE response because that is how it is
inactivated. Tyramine, however, is a catecholamine releasing agent. Cocaine would deplete
intracellular NE stores, thus reducing the effect of Tyramine.

Metoprolol is a somewhat selective B2>B1 blocker.


Nadolol is a non-selective beta blocker.
Carvedilol is a non-selective beta blocker and alpha-1 blocker.
Methyldopa inhibits DOPA Decarboxylase and thus the transformation of L-DOPA to dopamine,
and depletes catecholamines
Which of the following drugs will potentiate the cardiovascular actions of intravenously
administered norepinephrine but block the cardiovascular actions of intravenously administered
tyramine?

a) metoprolol
b) nadolol
c) carvedilol
d) methyldopa
e) cocaine
E

Therapeutic index = (LD50)/(ED50)


Margin of safety = LD1/ED99
The properties of drug A determined from quantal dose response curves are given
below. While drug A has several therapeutic effects, at certain concentrations it can result in
lethal effects. Calculate the Therapeutic Index for drug A
Drug A: ED50 3mg, ED99 = 100mg, LD1 = 200mg and LD50 = 300mg.
a) 0.3
b) 0.5
c) 2
d) 3
e) 100
C, but that is wrong. It should be 20 mg/L
Drug A is unionized, and fifty % of your patient blood is plasma. The blood concentration of drug
A is 17mg/liter. Calculate the concentration of drug in the plasma.
a) 2mg/Liter
b) 8.5mg/liter
c) 10mg/liter
d) 34mg/liter
e) 68mg/liter
B. 

Dm = (Cl Css Tm)/F
Your patient has been given a loading dose to achieve a therapeutic concentration of 10mg/liter.
Clearance of the drug is 1liter/8hours and the bioavailability of the drug is 0.5.
You have prescribed an oral preparation of the drug. Calculate the maintenance dose
that the patient will take to maintain the therapeutic concentration.
a) 20mg twice a day
b) 20mg three times a day
c) 40mg once a day
d) 40mg twice a day
e) 40mg three times a day
D

70% is absorbed, but 40% of the absorbed is metabolized, leaving 60% of what is absorbed to
be bioavailable. F = .7*.6 = .42
The drug you are administering to your patient is taken orally. The drug is 70% absorbed in the
gut, and 40% is metabolized in the liver. The bioavailability of the drug is:
a) 0.12
b) 0.18
c) 0.28
d) 0.42
e) 0.7
69. E
70. A
71. B
72. 

Tamusolin is an alpha-1 blocker


Carvedilol is a nonspecific beta blocker /alpha 1 blocker (reduces both cardiac output and
vascular resistance).
Metoprolol is a somewhat specific beta-1/beta-2 blocker
Phenoxybenzamine is an alpha blocker with somehwat selectivity for alpha-1, extremely long
acting
Phentolamine is an alpha blocker
For each patient described select the most appropriate drug therapy from the lettered choices
a) tamulosin
b) carvedilol
c) metoprolol
d) phenoxybenzamine
e) phentolamine

69. A woman experiences pain and ischemia in her finger after accidently injecting it with an
auto-injector that she carries for emergency treatment of allergic reactions

70. A man complains of urinary urgency, frequency and nocturia and is found to have
enlargement of the prostate gland

71. A woman with essential hypertension requires a drug that reduces both cardiac output
and peripheral resistance.

72. A man with episodic severe hypertension is found to have markedly elevated
epinephrine and norepinephrine metabolites in his urine and requires a long-acting drug
to lower blood pressure before surgery.
C. 50 liters
Vd = dose/Cp(0)
As the leader of the project testing a new drug, drug A you have carried out several test for the
new drug. You administer i.v.10 mg of drug A and get the graph shown below for the plasma
concentration vs time after drug administration.

Calculate the volume of distribution for drug A.


a) 10 liters
b) 25 liters
c) 50 liters
d) 250 liters
e) 500 liters
Error! Filename not specified.
A
As the leader of the project testing a new drug, drug A you have carried out several test for the
new drug. You administer i.v.10 mg of drug A and get the graph shown below for the plasma
concentration vs time after drug administration.

Calculate the half-life of the drug.


a) less than 5 hours
b) between 5 and 10 hours
c) more than 10 hours
d) can not be calculated, not enough information
e) the drug does not have a half-life
Error! Filename not specified.
E. ATP
Most, if not all, neurons contain multiple cotransmitters. This is true of both sympathetic and
parasympathetic nerves of the autonomic nervous system. A cotransmitter that is present in
both sympathetic adrenergic and parasympathetic cholinergic neurons is:

a) vasoactive intestinal polypeptide (VIP)


b) neuropeptide Y (NPY)
c) 5-hydroxytryptamine (serotonin)
d) dopamine (DA)
e) adenosine triphosphate (ATP)
B
A new β-adrenergic antagonist has been approved by the Food and Drug
Administration (FDA) for treatment of hypertension. This approval means that:
a) the drug has been found superior to currently available beta blockers
b) it has been tried in about 3,000 hypertensive patients and found better
than placebo
c) it has been tried in over 100,000 healthy volunteers and found to have no
adverse effects
d) its LD50 has been established in at least two species including humans
e) patients being treated with old fashion beta blockers should be switched to
the new drug

You might also like